Colorectal Flashcards

1
Q

Describe the grading system for haemorrhoids

A

Internal Hemorrhoids:
(Proximal to Dentate Line)
- Overlying Anoderm (Columnar Epithelium & Visceral Nerves)

Grading: Based on Prolapse Below Dentate Line
I (Primary): Prolapse Only with Strain
II (Secondary): Spontaneously Reduce
III (Tertiary): Manually Reduce
IV (Quaternary): Cannot Reduce

External Hemorrhoids:
Distal to Dentate Line
Overlying Skin (Squamous Epithelium & Somatic Nerves)

Mixed Hemorrhoids
Originate Both Above & Below the Dentate Line

How well did you know this?
1
Not at all
2
3
4
5
Perfectly
2
Q

List the non-excisional techniques available for management of haemorrhoids

A
  • Avoidance of constipation or diarrhoea
  • High fibre diet
  • Adequate daily fluid intake
  • Reduce time spent sitting on toilet advise don’t take phone or newspaper etc into loo with them
  • sitz baths
  • local anaesthetic gel analgesia
  • haemorrhoidal banding risks of band misplacement below dentate line = immediate pain treat by removing it. delayed pain, urinary retention and sepsis could indicate pelvic sepsis.
  • sclerotherapy by injecting small amount of 5% phenol in almond oil as irritatant that causes scarring and shrinking of haemorrhoidal tissue over time.
How well did you know this?
1
Not at all
2
3
4
5
Perfectly
3
Q

What are the surgical treatment options for haemorrhoids and what are their advantages/disadvantages?

A

HALRAR - Haemorrhoidal Artery Ligation and RectoAnal Repair - Where a proctoscope with doppler is used and guides suture placement to ligate the haemorrhoidal artery. Combined with mucopexy. Technically more challenging to learn, but equivalent results in terms of recurrence and less painful.

Closed Excision (Ferguson Technique)
Elliptical Excision of the Hemorrhoidal Bundle
Harmonic Shown to Have Decreased Pain and Bleeding than Monopolar
Deep Suture Ligation of the Vascular Pedicle at the Proximal Apex
Close with a Running-Locked Absorbable Suture
Consider Leaving the Distal End Open to Allow Drainage

Open Excision (Milligan-Morgan)
Elliptical Excision of the Hemorrhoidal Bundle
Do Not Close – Heals by Secondary Intention
No Difference in Results Between Open/Closed

Circumferential Stapled (Whitehead Hemorrhoidectomy)
By Transanal Circular Stapler
Only for Internal Hemorrhoids Proximal to the Dentate Line
Lower Pain & Recovery
Higher Recurrence & Incontinence Risk

How well did you know this?
1
Not at all
2
3
4
5
Perfectly
4
Q

How would you manage a patient with post-haemorrhoidectomy bleeding?

A

Simultaneously assess and resuscitate using CRISP algorithim.

Replace volume loss with blood if significant blood loss or haemodynamically unstable.

Ultimately patient needs to return to operating theatre to stop bleeding often requires suture if within the first 24 hours post surgery.

If >5 days down the track when eschar sloughs off, as long as patient stable will often settle without requirement for return to theatre.

How well did you know this?
1
Not at all
2
3
4
5
Perfectly
5
Q

What patients are at risk of post-haemorrhoidectomy bleeding and how can you mitigate some of these things?

A

Patient factors:
- On anticoagulants
- Haematological disorders
- Liver disease
(stop anticoagulants, don’t restart until >5 days post op, patient selection regarding the non-modifiable ones)

Disease factors:
- large grade 4 haemorrhoids

Technical factors:
- poor suture technique.
- Open haemorrhoidectomy higher risk of bleeding than closed.

How well did you know this?
1
Not at all
2
3
4
5
Perfectly
6
Q

What is pruritus ani and how is it worked up?

A

Itching/burning sensation around the anus.

Most common causes are:
Benign:
- Poor hygiene
- Idiopathic
- anorectal disease e.g haemorrhoids, fissures,
- Dietary e.g. caffeine, carbonated, spicy, dairy, tomatoes, citrus
- local irritation e.g. reaction to sanitary products or washing powder used on underwear
- infection
- lichen planus/ sclerosis
- dermatitis contact or radiation induced
- psoariasis
- diabetes
- CKD

Malignant:
SCC anus (bowens disease)
Pagets disease
Rectal cancer

Worked up with history and examination including proctoscopy. Skin biopsies may be required if any abnormality in skin appearance or concern for malignancy.

How well did you know this?
1
Not at all
2
3
4
5
Perfectly
7
Q

What is the rectosigmoid junction?

A

Junction where sigmoid becomes rectum denoted by the splaying of teniae coli, the sacral promontory, peritoneal reflection and is at level of S3.

How well did you know this?
1
Not at all
2
3
4
5
Perfectly
8
Q

What are Denovillier’s and Waldyers fascia?

A

Denovillier’s fascia is anterior to rectum (retrovesicular and restroprostatic in men, retrovaginal in women).

Waldyers fascia is posterior to rectum also called retrosacral

How well did you know this?
1
Not at all
2
3
4
5
Perfectly
9
Q

Describe the anatomy of the anus.

A

Anus extends from anorectal ring to anal margin and measures about 4cm. It is folded by 3 valves of Houston.

Anorectal ring corresponds to the levator ani muscle and is the superior margin of the surgical anal canal. (anatomical anal canal superior margin is dentate line)

The anus is divided by the dentate/pectinate line into upper 2/3 (columnar epithelium) and lower 1/3 (squamous epithelium). This is also where the crypts of Morgagni originate.

The introitus of the anal opening is called the anal verge and the anal margin extends radially 5cm out from the anal verge into the perianal skin.

The anus has an internal and external sphincter with intersphincteric groove indicated by white line landmark.

The internal sphincter is a continuation of muscularis propria and is smooth muscle and involuntary control by pelvic splanchnic nerves.

The external sphincter is striated muscle under voluntary control by internal pudendal nerve (inf rectal br) and S4 (perineal br.).

The blood supply by inferior rectal artery from internal iliacs via internal pudendal artery. The venous drainage follows artery into inferior rectal vein to internal iliac and inferior vena cava.

The lymphatic drainage anal canal to internal iliacs, anal verge to inguinal.

How well did you know this?
1
Not at all
2
3
4
5
Perfectly
10
Q

What is a HAL-RAR

A

Stands for Haemorrhoidal Artery Ligation and RectoAnal Repair.

It is a procedure used to treat grade 3/4 internal haemorrhoids.

It involves using an USS doppler to locate the haemorrhoidal artery and ligating around this with a suture (HAL) and a mucopexy or mucopexies are performed in a running suture (always culminating at least 5mm above dentate line) and then being pulled up with a slip knot to the origin to pexy the prolapsed mucosa back up into anal canal.

https://www.amisurgical.com/ami-hal-rar

How well did you know this?
1
Not at all
2
3
4
5
Perfectly
11
Q

What is the Cryptoglandular theory?

A

The anal glands/crypts of Morgagni at the dentate line become blocked by debris resulting in build up, nidus for infection to brew and then turns into an abscess.

How well did you know this?
1
Not at all
2
3
4
5
Perfectly
12
Q

What are the spaces where an anorectal abscess may arise? And how might these present?

A
  • Perianal - subcutaneous around anal verge.

Submucosal - submucosal above dentate line

  • Ischiorectal within fat of ischiorectal fossa, tender away from the anal verge, DRE not particuarly painful.
    > post anal = posterior between levators and external anal sphincter. Can be superficial or deep to anococcygeal ligament.
    > horseshoe = extends bilaterally
  • Intersphincteric between internal and external sphincters. Painful DRE but few external findings.
  • Supralevator - Can really only evaluate on imaging e.g. CT or MRI.
How well did you know this?
1
Not at all
2
3
4
5
Perfectly
13
Q

How do you go about draining a perianal abscess, submucosal abscess, ischiorectal abscess, intersphincteric abscess and supralevator abscess?

A

Perianal - Examine preop and confirm w patient where most tender and mark with permanent marker over most tender and fluctuant area. EUA, positioned lithotomy, good lighting. DRE +/- rigid sigmoidscopy. New gloves. Radial incision oriented over side closest to anal margin and drain cavity.

Submucosal abscess

Ischiorectal abscess similar to perianal except may need to puncture anococcygeal ligament if deep and may need bilateral incisions if horseshoe (see Hanley procedure)

Intersphincteric - requires transanal drainage through incision in internal sphincter.

Supralevator: Need to drain the source…
Intraabdominal source - transabdo surgery or perc drain
- Intersphincteric Fistula: Transanal Drainage (Divide IAS)
- Transsphincteric/Ischiorectal Source Through Levators: Ischiorectal I&D

How well did you know this?
1
Not at all
2
3
4
5
Perfectly
14
Q

What are the indications for antibiotics in anorectal abscess’?

A

Most do not require antibiotics post op. I would still tend to prescribe them pre-op while awaiting access to the acute theatre.

Those I would prescribe them for post-operatively would be those with cellulitis or systemic sepsis, diabetes or immunocompromised. Also those with metallic heart valves.

How well did you know this?
1
Not at all
2
3
4
5
Perfectly
15
Q

Describe Goodsall’s law

A

Anterior fistula tracts drain radially to their internal opening to the nearest crypt in a linear path unless >3cm away from anal verge in which case can drain like a posterior fistula via long curved path to posterior midline.

Posterior fistula tracts drain towards posterior midline and take a curved path.

How well did you know this?
1
Not at all
2
3
4
5
Perfectly
16
Q

Describe the different locations of anal fistula.

A
  • Submucosal - Superficial between perianal skin and anal mucosa
  • Intersphincteric between perianal skin, traverses internal sphincter but not external sphincter and terminates in anal mucosa
  • Transphincteric between perianal skin, tranverses both internal and external sphincters and terminates in anal mucosa.
  • Suprasphincteric between perianal skin, through levator ani and down into intersphincteric groove then traverses internal sphincter and into anal mucosa.
  • Extrasphincteric between perianal skin, through levator ani and terminates in rectal mucosa more superiorly.
How well did you know this?
1
Not at all
2
3
4
5
Perfectly
17
Q

How does botox work for chronic anal fissures?

A

The pathogenesis of chronic anal fissures is hypertonia of the interal anal sphincter and resultant reduced blood flow to the anal skin and mucosa and leads to an ischaemic cycle and therefore reduced ability to heal.

The botullinum toxin works as a neuroblockade preventing release neurotransmitter acetylcholine from axon endings at neuromuscular junction and in doing so results in relaxation of the internal sphincter. In relaxing the sphincter it allows blood flow to resume to the skin and allow for better healing.

How well did you know this?
1
Not at all
2
3
4
5
Perfectly
18
Q

Describe a treatment ladder for anal fissure.

A

Starting with the least invasive/fewest side effects the patient would be advised to increase fluid intake, increase fibre in diet, avoiding constipation, avoiding straining or long periods sitting on the loo so stool softeners may be required. Sitz baths.

The next step I prescribe are topical treatments. For analgesia lidocaine gel. To try reverse the hypertonia of the internal sphincter I prescribe diltiazem 2% ointment that needs a compounding pharmacy to make it. This is a calcium channel blocker. I prefer this as less likely to give a headache over glyceryl trinitrate cream 0.2% which is an alternative. With both of these need to take care in patients with cardiac comorbidities.

Should these treatments fail then under anaesthesia injection of botox is recommended. I use a lateral sphincterotomy as last resort and am mindful of contraindications if any concerns for faceal incontinence already then these should not be attempted or if on use of botox the patient gets faecal incontinence then they should not proceed to sphincterotomy.

Lastly mucocutaneous advancement flaps can be used to cover over the fissure to facilitate healing.

Another relative contraindication to surgical procedures is in Crohns unless absolutely necessary.

How well did you know this?
1
Not at all
2
3
4
5
Perfectly
19
Q

What are the most common pathogens in perianal abscess’?

A

Most common gut microbiota found in perianal abscesses are Escherichia coli, Proteus vulgaris, Staphylococcus aureus, Streptococcus species, Bacteroides, and Peptostreptococcus species

How well did you know this?
1
Not at all
2
3
4
5
Perfectly
20
Q

Classify the different types of ano-rectal incontinence

A

Urge incontinence - sensate but unable to retain faecal material despite active attempts. Usually points to sphincter dysfunction or inability of rectum to hold on to stool.

Insensible incontinence - insensate passive incontinence indicates neurogenic cause, sphincter dysfunction or impaired anorectal reflexes

Faecal seepage - leakage post a bowel motion but retained continence.

How well did you know this?
1
Not at all
2
3
4
5
Perfectly
21
Q

What are the treatments for faecal incontinence?

A

Fecal incontinence is a complex issue that is not easy to manage. The vast number of methods used to manage the condition indicates that no method works reliably. It requires a multidisciplinary approach including mental health support.

Lifestyle:
- weight loss
- dietary changes to reduce caffeine
- high fibre diet

Medication
- fibre supplements
- stool softeners and laxatives for chronic constipation as underlying cause
- anti-diarrhoeal medication e.f loperamide, codeine for underlying diarrhoea as cause
- oestrogen relacement treatment may help post menopausal
- amitryptylline for faecal and urinary incontinence

Physio therapy:
- Exercises to strengthen pelvic floor
- electrode stimulation to contract pelvic floor in those that are unable to do so voluntarily due to pudendal nerve damage
- sacral nerve stimulation. Not publicly easily accessible in NZ.

Surgery
- those without intact sphincter or who have failed above management consider surgery.
Options include sphincteroplasty, post anal augmentation to recreate anorectal angle, reconstruction with a neosphincter e.g. using gracilis muscle
Repair of underlying cause e.g haemorrhoids, prolapse
- faecal diversion with stoma

https://www.ncbi.nlm.nih.gov/books/NBK459128/#:~:text=Urge%20Incontinence%3A%20Inability%20to%20retain,bowel%20movement%20with%20normal%20continence

How well did you know this?
1
Not at all
2
3
4
5
Perfectly
22
Q

What is demonstrated here and how would you treat it?

A

This is a defecating proctogram showing resting, squeezing, straining and defecating views. It is a fluoroscopy study to assess for ano-rectal function and to diagnose rectocoele and prolapse.

This one demonstrates a rectocoele. This would be treated with lifestyle modification with increased fluid and fibre, weight loss, physiotherapist and biofeedback and in those refractory to above then surgery can be considered via transanal or transvaginal approach.

How well did you know this?
1
Not at all
2
3
4
5
Perfectly
23
Q

What diagnostic tools are available for ano-rectal incontinence?

A

History and Examination:
- assess for anal tone
- anorectal descent
- sensation
- anal wink reflex (perianal skin with a cotton ball, which causes brisk contraction of the external anal sphincter. The absence of this reflex indicates a loss of spinal arc and possibly underlying neurological disease.)

Laboratory:
- Stool spec for diarrhoeal pathogens
- Faecal elastase to assess for Pancreatic insufficiency

Endoscopic:
- Colonoscopy to evaluate for IBD or malignancy

Imaging:
- Endoanal ultrasound - assess’ the thickness of IAS and EAS
- MRI
- Defecating proctogram (fluroscopic images obtained during defecation)

Other:
- anal manometry to assess the resting and squeeze rectal pressure; the technique can also be used to assess rectal capacity and compliance
- Measuring pudendal nerve latency: To assess the integrity of the pelvic floor neuromuscular integrity

How well did you know this?
1
Not at all
2
3
4
5
Perfectly
24
Q

How does biofeedback work for faecal incontinence?

A

It works on concept of cognitive retraining of the pelvic floor and abdominal musculature to overcome the defects. It is indicated for those that have shown on anal manometry to have impaired external sphincter tone and loss of sensation to rectal distension.

How well did you know this?
1
Not at all
2
3
4
5
Perfectly
25
Q

What is sacral nerve stimulation and how is it performed?

A

The 2-step procedure involves initially placing temporary external electrodes into the sacral foramen. The stimulation decreases symptoms of fecal incontinence by enhancing the squeeze and resting anal pressures and colonic motility. Patients who respond then undergo permanent placement of an embedded neurostimulator. While good outcomes have been reported in several studies, the surgery can be associated with hematoma, seroma, and infection. In addition, lead migration and paresthesias are not uncommon. To counter these problems. sacral transcutaneous electrical nerve stimulation is now being evaluated.

How well did you know this?
1
Not at all
2
3
4
5
Perfectly
26
Q

Describe ways of classifying anal fistulas

A

Depth: High vs low (<30% transphincteric)

Complexity: Simple (single and low) vs complex (high or multiple)

By their anatomic location:(superficial, intersphincteric, transsphincteric, suprasphincteric, extrasphincteric)

How well did you know this?
1
Not at all
2
3
4
5
Perfectly
27
Q

Describe how you would place a seton.

A

In an appropriately worked up and consented patient I would position the patient in lithotomy with bright lighting. I would perform a digital rectal examination and proctoscopy. I would use a parks retractor with ample lubricant. I would use a Lockhart Mummery probe placing it into the external opening first so as not to inadvertently create false passages. Using Goodsall’s law I would look for the internal opening in the expected position. If the internal opening is not clear with these methods then using a syringe, an IVL catheter and hydrogen peroxide I can try locate the internal opening by flushing this in through external opening and observing for bubbles in anal mucosa. Once tract established I would feed a seton (vascular loop) on a mosquito clip through the external opening, retrieve it at internal opening and secure it with silk ties. I would apply this loose at skin. I would then leave this for minimum 6 weeks, often longer for tract to mature, dry

How well did you know this?
1
Not at all
2
3
4
5
Perfectly
28
Q

If setons fail or as part of a staged approach with setons what are the surgical options available for anorectal fistulas?

A
  • Fistulotomy only if superficial enough.
  • LIFT procedure
  • Bio LIFT
  • Mucosal or cutaneous advancement flap
  • Fibrin glue
  • Fistula collagen plug
How well did you know this?
1
Not at all
2
3
4
5
Perfectly
29
Q

What medical management options are available for anorectal fistulising Crohns disease?

A
  • Immunomodulators e.g. 6-mercaptopurine and azathioprine
  • Aminosalicylates e.g. sulfasalazine
  • Methotrexate
  • Inflixumab - anti TNFa chimeric monoclonal antibody. Other anti-TNFa therapies also available.
  • Immunosuppresants e.g. cyclosporin, mycophenylate mofetil
    Antibiotics - Ciprofloxacin and Metronidazole combo treatment, tacrolimus
    Corticosteroids - not so much for fistulising Crohns.

https://www.gastrojournal.org/article/S0016-5085(00)80030-X/fulltext

How well did you know this?
1
Not at all
2
3
4
5
Perfectly
30
Q

What is Hirschsprungs disease?

A

Basics

Absence of Intramural Ganglion Cells of the Meissner (Submucosal) & Auerbach (Myenteric) Plexuses
From Defect in Craniocaudal Neuroblast Migration
Most Commonly Less Severe & Affects Only a Short Segment
Rare in Adults, 94% Diagnosed Before Age 5 Years
Associated with RET Mutation
Seen Especially with Long-Segment Disease – Consider MEN IIA Evaluation
More Common in Males
Presentation

Lifelong History of Chronic Constipation
Abdominal Pain
Abdominal Distention
Diagnosis

Dx: Full Thickness Rectal Bx with Aganglionosis
Radiology: Grossly Dilated Proximal Colon with Fecal Retention
Treatment

Treatment: Surgery
Short-Segment: Transanal Myomectomy
Long-Segment:
Swenson Procedure
Rectosigmoid Resection & Colorectal Anastomosis
Duhamel Procedure
Healthy Colon Lowered Posterior to Diseased Rectum Through the Avascular Plane
Diseased Rectum Left Intact
Soave Procedure
Resect Diseased Colon & Upper Rectum
Mucosa of Diseased Lower Rectum Resected
Colon Pulled Through the Lower Rectum

How well did you know this?
1
Not at all
2
3
4
5
Perfectly
31
Q

You see this on colonoscopy on anterior rectal wall. What are you differentials?

A

Benign:
Solitary rectal ulcer syndrome due to prolapse. Confirm with history, examination and biopsy.

Proctitis

Malignant:
Site of previously excised polyp/malignancy. Confirm with history, examination and biopsy.

How well did you know this?
1
Not at all
2
3
4
5
Perfectly
32
Q

What are the risk factors for pilonidal disease?

A
  • Young (teens- 20s)
  • Male > Female
  • Hirsute (thick hairy)
  • Obese
  • Deep gluteal cleft
  • Caucasian
  • Sitting for long periods >6h
  • Local trauma
How well did you know this?
1
Not at all
2
3
4
5
Perfectly
33
Q

Describe a classification system for pilonidal disease.

A

There are many out there!

I use Guner classification as easier to remember given this disease’s association with Jeep drivers in the world war ‘Gunner’.

Stage 1: single midline pit
Stage 2: Multiple pits in midline
Stage 3: Midline pits with lateral extension in one direction
Stage 4: Midline pits with lateral extension bilaterally
Stage R: Recurrent disease post operative intervention

Other classifications. include:
Tezel Classification looking at acuity of the disease
Chavoin Classification
Doll Classification
Irkorucu Classification
Karkas Classification
Lapsekili Classification
Awad Classification

How well did you know this?
1
Not at all
2
3
4
5
Perfectly
34
Q

How do you treat acute abscess associated with pilonidal disease?

A

Drain the pus through an incision lateral to the cleft. Heals better. Currettage or tract excision in acute disease is not successful. However simple drainage as above is successful final treatment for 60%.

Include antibiotics if systemically unwell, cellulitis, diabetic or immunocompromised.

How well did you know this?
1
Not at all
2
3
4
5
Perfectly
35
Q

What are the principles of elective surgery for pilonidal disease and describe some examples of how these priniciples are achieved.

A

Priniciples are predominantly to
- excise pits
- remove trapped hair
+ if performing flap surgery:
- flatten out gluteal cleft
- closure off midline

Examples are:
Minimally invasive options include:
- Pit picking - with punch biopsies of pits and removal of trapped hair with fine clamps
- video assisted sinus ablation
- midline pit excision
- marsupialisation
- excision plus flap reconstruction to flatten gluteal cleft

How well did you know this?
1
Not at all
2
3
4
5
Perfectly
36
Q

What are the differentials for causes of proctitis and how are they treated?

A

Infectious:
- most common in MSM
- gonnorhoea and chlaymida most common treated with ceftriaxone and doxycylcine respectively
- other causes HSV (acyclovir) and syphllis (penicillin)

Radiation induced
- acute usually within 6 weeks of treatment
- may require cessation of rtx
- hydration
- antidiarrhoeals
- butyrate enemas
- sucralfate enemas

chronic = obliterative arteritis (months-yrs)
- sucralfate enemas
- APC
- last resort surgery

Pouchitis after ileal ouch-anal anastomosis

Diversion proctitis

Neutropenic enterocolitis (Typhilitis) but rectal involvement v rare

How well did you know this?
1
Not at all
2
3
4
5
Perfectly
37
Q

Differentials go

A
  • Anal warts secondary to HPV.
    (Condyloma acuminata)
  • Anal SCC
  • Pagets disease
How well did you know this?
1
Not at all
2
3
4
5
Perfectly
38
Q

Which HPV strains are high risk and which are low risk for malignancy?

A

High risk 16 & 18
Low risk 6, 11

Other high risks
31,33,35,39,45,51,52,56,58
Other low risks:
42,43,44

How well did you know this?
1
Not at all
2
3
4
5
Perfectly
39
Q

How do you treat anal condylloma acuminata?

A

Goal: Remove Macroscopic (Visible) Disease

At-Home Tx:
Topical Tx: Podofilox, Imiquimod or 5-Fluorouracil (5-FU)
40-60% Clearance Rate

Office-Based Tx:
Topical Tx: Podophyllin or Trichloroacetic Acid (TCA)
Liquid Nitrogen Cryotherapy
80-90% Clearance Rates

Surgical Tx: Electrocautery/Fulguration or Excision
Indications: Extensive External Disease or Lesions in Anal Canal
If Circumferential: Staged Surgery (Prevent Stricture)

If histology shows Dysplasia this does NOT require Wide Excision – Managed Similarly to Non-Dysplastic Condyloma.
High Recurrence Rates with Unknown Benefit in Prevention

How well did you know this?
1
Not at all
2
3
4
5
Perfectly
40
Q

Spot diagnosis go

A

Buschke-Lowenstein tumour/giant condyloma

  • rare, large, slow growing anal wart with 50% risk of malignancy.
    Needs wide local excision, high recurrrence rate and may need APR.

(think must be living in the bush (buschke) to present that late.

How well did you know this?
1
Not at all
2
3
4
5
Perfectly
41
Q

How do you diagnose a rectovaginal fistula?

A

History (esp take note of obstetric trauma, crohns, malignancy, surgery, diverticulitis, radiation), any gas or faeces from vagina, dyspareunia, recurrent UTIs

Examination - any faeces in vagina? any visible tract on speculum on proctoscopy.
EUA

Methylene blue tampon test, methylene blue enema in rectum, clean tampon in vagina, ambulate then assess tampon for any blue.

Endoscopy make sure to retroflex in rectum

Fluroscopic xray study or CT with rectal contrast

How well did you know this?
1
Not at all
2
3
4
5
Perfectly
42
Q

What is AIN and how is it managed?

A

Definition: Anal intraepithelial neoplasia is a dysplasia premalignant for anal SCC.

Incidence/risk factors:
- HPV + below
- HIV or immunocompromised incl solid organ transplant recipients
- MSM or anoreceptive sex
- smoking
- HPV‐related gynecological precancerous lesions or cancer (vulvar, cervical, or vaginal)

Pathogenesis:
HPV induced chronic inflammation induces dysplastic changes in epithelial cells that progress to squamous cell carcinoma if left unchecked.

Signs and symptoms:
Assymptomatic to perianal skin irritation or wetness or pruritis or bleeding.

Classification system:
LSIL: Low grade intrepithelial lesion = replacement of lower third of epithelium with abnormal cells
HSIL = replacemen of middle and upper thirds of epithelium with abnormal cells.

Investigations:
- EUA
- punch biopsies
use dilute acetic acid painted on to skin to identify areas to biopsy, if no areas visible then mapping biopsies in 4 quadrants.
- women also need cervical smears uptodate

Treatment: All should receive HPV quadrivalent vaccine as well.
LSIL: Observation
Surveillance Every 3-6 Months
Consider High Resolution Anoscopy (HRA) to Surveil
HSIL: Topical Therapy & Consider Ablation
Topical Tx: Imiquimod, 5-Fluorouracil (5-FU) or Trichloroacetic Acid (TCA)
Ablation Tx: Radiofrequency Ablation (RFA) or Electrocautery

How well did you know this?
1
Not at all
2
3
4
5
Perfectly
43
Q

What is the TNM staging of anal canal cancer?

A

T1 = <2cm
T2 = 2-5cm
T3 = >5cm
T4 = invading other organs

N1 = nodes involved
M1 = mets present

Stage 1 & 2 are all N0/M0
- stage 1 = T1
- stage 2a = T2
- stage 2b = T3

Stage 3a = T1/2, N1, M0
Stage 3b = T4, N0, M0
Stage 3c = T3/4, N1, M0

Stage 4 is any T any N M1

How well did you know this?
1
Not at all
2
3
4
5
Perfectly
44
Q

What is the treatment for anal SCC?

A

Nigro protocol
Chemoradiation with mitomycin and 5 flurouracil with reassessment at 8-12 weeks then again every 6-8 weeks until completion.

If fails after 6m, confirm with biopsy then APR unless if <2cm could consider WLE w 1cm margin.

How well did you know this?
1
Not at all
2
3
4
5
Perfectly
45
Q

Who should be considered for screening for anal cancer?

A

We don’t have a screening program in NZ but those that are at high risk should be aware of this and what to look out for. People at high risk include
- MSM
- prev precancer or cancer of vulva/vagina/cervix
- high risk HPV strain (16 &18)
- immunocompromised e.g. HIV or solid organ transplant recipients

How well did you know this?
1
Not at all
2
3
4
5
Perfectly
46
Q

What are the most common anal canal cancers?

A

SCC - most common ~85% Nigro
Adenocarcinoma ~10% WLE/APR
BCC
Melanoma (often mistaken for thrombosed haemorrhoid) NB no SNB required.

How well did you know this?
1
Not at all
2
3
4
5
Perfectly
46
Q

What is high resolution anoscopy?

A

Anal examination using proctoscope, colposcope and acetic acid. Allows for magnified look at anal epithelium to assess for SCC.

How well did you know this?
1
Not at all
2
3
4
5
Perfectly
47
Q

What is perianal Pagets disease?

A

Intraepithelial Adenocarcinoma of perianal skin.

usually presents as severe intractable pruritis ani and erythematous eczematous rash

Patients need colonoscopy as work up to look for synchronous colorectal malignancy (~50% pts have a synchronous visceral carcinoma) but can arise as primary pagets from apocrine glands without colorectal tumour.

How well did you know this?
1
Not at all
2
3
4
5
Perfectly
48
Q

What imaging is required in staging a rectal cancer and what are they each used for?

A

EUS - most useful for accurate T staging

MRI - best at assessing mesorectal fascia involvement and circumferential margin and nodes

CT - for distant mets.

How well did you know this?
1
Not at all
2
3
4
5
Perfectly
49
Q

What is the T staging for rectal cancer?

A

Follows that of colon cancer not anal cancer.

Stage 1-2 N0/M0
Stage 4 any T any N M1

How well did you know this?
1
Not at all
2
3
4
5
Perfectly
50
Q

What are the treatment options for low risk rectal cancer?

A

Low risk rectal cancer is defined as:
- T1/2
- <3cm
- <30% of circumference
- <8cm from anal verge
- mobile
- no lymphatic invasion
- no mucin production

These may be amenable to endoscopic or surgical resection.

If endoscopic required 10mm margin. Does not harvest any lymph nodes.

If resection options include low anterior resection or abdominoperineal resection. Need 5cm proximal margin, 1-2cm distal margin. Goal lymph node harvest 12+.

How well did you know this?
1
Not at all
2
3
4
5
Perfectly
51
Q

What are the treatment options for locally advanced rectal cancer?

A

Locally advanced rectal cancer includes those with:
- T3+ disease
- Nodal involvement
- Lymphovascular invasion
- >3cm
- >30% of circumference
- >8cm from anal verge
- tethered
- mucin production

Treatments are neoadjuvant chemoradiation therapy with 5 flurouracil or FOLFOX or CAPEOX followed by resection ~2-3months later.

Adjuvant therapy with 5FU, FOLFOX or CAPEOX in threatened CRM in T3 disease or for T4 or N+.

How well did you know this?
1
Not at all
2
3
4
5
Perfectly
52
Q

What are the layers of the colon wall?

A
  • Mucosa (columnar epithelium, lamina propria, muscularis mucosa)
  • Submucosa
  • Mucularis propria (inner circular, outer longitudinal)
  • Serosa
How well did you know this?
1
Not at all
2
3
4
5
Perfectly
53
Q

Describe the anatomy of the colon.

A
  • Colon is ~155-165cm long.
  • It is divided into caecum, asc, transverse, desc and sigmoid portions.
  • It has 3 taenia coli (longitudinal strips of muscle that coalesce at the appendix and splay out over the rectum.
  • the caecum, asc and desc portions are retroperitoneal, transverse and sigmoid are intrabdominal.
  • the diameter of colon ranges from 3-9cm with caecum being the largest part at 9cm and is therefore place of highest risk for non-iatrogenic perforation (due to LaPlaces law (Largest radius will have highest surface tension)
  • Wall is made up of mucosa, submucosa, muscularis propria (IC, OL) and serosa.
  • it is folded into haustra by plicae semiluminares
  • Its attached to the omentum along transverse colon.
  • its relations include small bowel, kidneys and ureters, gonadal vessels, liver and spleen, stomach, pancreas and lesser sac.

Vascular supply:
- SMA > ileocolic, right colic & middle colic
- IMA > left colic, sigmoid and upper rectal arteries.
Hence watershed areas are splenic flexure and between upper and middle rectum.

Drains to IMV which joins SMV and becomes portal vein behind the pancreas.

Lymphatics follow arteries > SMA/IMA > preaortic > cisternia chyli.

Innervation:
- Meissners submucosa plexus
- Auerbachs myenteric plexus, outer muscularis propria
- Pelvic splanchnic nerves parasympathetic

Functions to secrete K and resorb NaCl/H2O with largest H2O capacity in the asc colon.

How well did you know this?
1
Not at all
2
3
4
5
Perfectly
54
Q

What is the Paris classification of colonic polyps?

A

Describes their appearance.

Polypoid:
Type 1p - pedunculated (on a stalk)
Type 1s - sessile (base same diameter as top)

Non-polypoid:
Type 2a - slightly elevated
Type2b - flat (height <1/2 diameter(
Type2c - depressed

Excavated:
Type 3:Cratered

How well did you know this?
1
Not at all
2
3
4
5
Perfectly
55
Q

What are the histological subtypes of colonic polyps?

A

Hyperplastic - normal cellular components with proliferative characteristics but no dysplasia.
- classic saw tooth pattern

Adenomas -
-tubular adenoma = >75% tubular features with network of branching adenomatous epithelium. most common neoplastic polyp. usually pedunculated.

  • villous adenoma = >75% villous features with long glands extending from the surface straight down. most likely to produce symptoms. 50% have cancer.
  • tubulovillous >25% both tubular and villous features.

Hamartomas = disorganised growth of normal tissue elements. Benign but can degenerate to adenomatous.

How well did you know this?
1
Not at all
2
3
4
5
Perfectly
56
Q

What are the depth of invasion classification systems for colonic malignant polyps?

A

Level 0 - does not invade muscularis mucosa (in situ)
Level 1 - invades hed
Level 2 - invades neck
Level 3 - invades stalk
Level 4 - invades base or involved ina sessile polyp i.e. all sessile polyps with invasive cancer are level 4.

Risk of lymph node metastasis =
<1% for Levels 0-3
30% for Level 4

SM1 - Superficial third
SM2 - Middle third
SM3 - Deep third

Risk of lymph node metastasis =
SM1 <2%, SM2/3 22-33%

How well did you know this?
1
Not at all
2
3
4
5
Perfectly
57
Q

What are some genetic syndromes associated with increased risk of colorectal cancer?

A
  • Familial Adenomatous Polyposis (FAP) mutation in APC gene close to 100% chance of developing ca.
  • MUTYH-Associated Polyposis (MAP) mutation in MUTYH gene~85%
  • Lynch syndrome also known as Hereditary Non-Polyposis Colon Cancer (HNPCC) (MLH1, MSH2, MSH6, PMS2, EpCAM mutations) 20-80%
  • Serrated Polyposis Syndrome (genetic mutation largely unknown but closest link is with RNF43) 25-75%
  • Juvenille Polyposis Syndrome (SMAD4, BMPR1A mutations) 68%
  • Peutz-Jeghers (STK11 mutation) 39%
  • PTEN Hamartomas syndrome (PTEN mutation) 9%

NB: FAP is only one where prophylactic colectomy is advised.

All are autosomal dominant inheritance except MAP (AR) and SPS where no clear consistent genetic mutation isolated.

How well did you know this?
1
Not at all
2
3
4
5
Perfectly
58
Q

What is the diagnosis? Tell me about this syndrome.

A

Familial Adenomatous Polyposis Syndrome (FAP)

Hallmark is 100-1000s of sessile polyps.

Incidence ~1/8300 births

Lifetime risk of developing colorectal ca 100%

Onset of polyps ~50% developed by 15yo, 95% by 35yo. Av age of ca dx 35-40yo. Most of these are left sided.

Each individual polyp not at an increased risk of developing ca but the sheer abundance is why such an increased risk.

Extracolonic manifestations:
- Duodenum polyps (graded with Spiegelmans staging system looking at number of polyps, size, histological subtype & dysplasia). 2nd most common site of polyps for FAP, 2nd most common cause of death for FAP pt w predilection for periampullary region.
- Stomach polyps usually benign fundic gland polyps.
- Desmoids locally advanced 3rd most common cause of death in FAP
- Osteomas
- Brain tumours e.g. medulloblastoma
- Congenital Hypertrophy of Retinal Pigment Epithelium (dark spots on retina). Most common and earliest extracolonic manifestation of FAP. Benign.
- Dental problems e.g. missing teeth, extra teeth, impacted teeth etc.
- less common include thyroid ca, hepatoblastoma, adrenal tumours, pancreatic ca…

Genetic counselling and screening recommendations:
- Those with family hx of FAP should undergo genetic counselling and screening by age 10-12yo
- yearly colonoscopy from age 10-15yo
- upper endoscopy age 20-25 or sooner once any colonic polyps are seen. Use Spigelman stage to define frequency of screening ranging from Stage 4 every 3-6m to Stage 0 every 3-5 years.
- May also consider USS thyroid screening from late teens.

Colectomy indicated in almost all patients as treatment or prophylaxis but exact timing is debatable. Most are around age 15-25yo. Nuances of selection see another flash card.

How well did you know this?
1
Not at all
2
3
4
5
Perfectly
59
Q

What are the surgical options and indications for FAP patients?

A

2015 ACG Guidelines
Therapeutic colectomy:
———————————-
Absolute indications:
- Documented colorectal cancer
- significant symptoms

Relative indications:
- multiple adenomas >6mm
- significant increase in adenoma numbers on serial exams
- adenoma with HGD
- inability to adequately survey colon due to multiple diminuitive polyps

If otherwise not indicated above but exact timing is debatable. Most will be age 15-25yo.

The options include:
Total Abdominal Colectomy with Ileal-rectal anastomosis (i.e. spares rectum)
Adv: technically easier, lower risk of complications, no risk of sexual and bladder dysfunction, no permanent stoma.
Disadv: risk of metachronous ca in rectum 5-25%. Generally contraindicated for severe rectal disease or if patient not reliable for follow up surveillance. Requires flexible proctoscopy every 6-12m.

Total proctocolectomy and Ileal pouch-anal anastomosis.
Adv: lower risk of metachronous ca, no permanent stoma
Disadv: technically more challenging, higher risk of complications, risk of sexual or bladder dysfunction. Requires flexible pouchoscopy every 6-12m.

Total proctocolectomy with end ileostomy.
Adv: Lowest risk of metachronous ca.
Disadv: Requires permanent stoma. Risks sexual and bladder dysfunction. Requires flexible endoscopy in ostomy every year.

How well did you know this?
1
Not at all
2
3
4
5
Perfectly
60
Q

What is the most common cause of hereditary colorectal cancer? Tell me about the syndrome.

A

Lynch syndrome otherwise known as Hereditary Nonpolyposis Colon Cancer (HNPCC).

Mutations:
Most common mutations are inherited in autosomal dominant pattern with mutations in Mismatch repair (MMR) genes: MLH1- MSH2, MSH6, PMS2 or in epCAM a mutation in which causes epigenetic silencing of MSH2.
Leads to microsatellite instability.

Incidence:
Accounts for 3-5% of all colorectal ca patients. Younger onset than sporadic cancers av age 45-60.
1/279 births. 20% are sporadic mutations.

Location:
- Most commonly ca on right colon (c.f. sporadic ca’s most commonly on left)
- Propensity of synchronous and metachronous cancers.

Types:
Type 1 - no extracolonic manifestations
Type 2 - colonic + extracolonic manifestations

Extracolonic manifestations:
- Endometrial ca
- Ovarian ca
- Cancers of KUB
- Stomach
- Pancreatic
- HPB
- Breast
- Brain
- Skin

Clinical diagnosis criteria:
Amsterdam II (remember 3,2,1 rule)
- ≥ 3 Relatives with Histologically Verified Lynch Syndrome-Associated Cancer, One of whom is a first-degree relative of the other two
- ≥ 2 Generations involved
- ≥ 1Cancer case was dx before age 50
- FAP excluded.
Sensitivity 78% and Specificity 61%

Screening recommendations:
- Colonoscopy age 20-25yo
- Repeat every 1-2 years
- Upper endoscopy age 30-40 every 2-4 years
- Consider endometrial biopsy froma age 35 every 1-2 years but no proven benefit

Therapeutic Colectomy

Indications: Colorectal Cancer or Unresectable Adenomas
Colon Cancer: Total Abdominal Colectomy with Ileorectal Anastomosis (TAC/IRA) is Generally Preferred but may consider segmental colectomy in select circumstances.
Requires surveillance with Flexible Proctoscopy or Repeat Colonoscopy annually respectively.

Rectal Cancer: Consider Total Proctocolectomy (w end colostomy or ileal pouch anal anastomosis) vs Segmental Resection (low anterior resection or APR)

Prophylactic Colectomy
- generally not performed.
some indications:
- Colon technically difficult to navigate
- Unable to comply with screening
- Severe psychological distress due to fear of developing CRC
- Families with early onset or severe penetrance of CRC
- Females already undergoing Hysterectomy for Endometrial ca or a Prophylactic Total Hysterectomy and Bilateral Salpingo-Oophorectomy (TH-BSO) in females after age 40 or once child bearing completed.

How well did you know this?
1
Not at all
2
3
4
5
Perfectly
61
Q

Tell me about Juvenile Polyposis syndrome.

A

Mutation in SMAD4 or BMPR1A genes with Autosomal dominant inheritance.

Juvenile refers to appearance of the polyp not the age of onset though most present symptomatic with GI bleeding and anaemia by age 20.

Polyps are Hamartmatous and found mostly through colon (98%) but also stomach, duodenum and rest of small intestine.

Most hamartomas are benign but malignant transformation can occur.

Extraintestinal manifestations include:
Hereditary Haemorrhagic telengectasia, epistaxis and pulmonary AVMs. Skeletal stigmata include macrocephalus, hydrocephalus, cleft palate, polydactyly and hypertelorism

How well did you know this?
1
Not at all
2
3
4
5
Perfectly
62
Q

Which colorectal cancer syndrome has autosomal recessive inheritance? Tell me about that syndrome.

A

MUTYH associated polyposis (MAP)

Develop 10s-100s of adenomatous polyps, predominantly right sided. Can also have increased number of hyperplastic and mixed polyps.

Increased risk of colorectal ca.
- ~60% by age 60
- 80-90% lifetime risk without surveillance.

Extracolonic manifestations:
Other GI tract polyps and cancers
-Duodenal Polyps (17-34%)
-Duodenal Cancer (4%)
-Gastric Polyps (11%)
-Gastric Cancer (1% – Not Significantly -Higher Than the General Population)
-Bladder Cancer
-6-8% in Females
-6-25% in Males
-Ovarian Cancer (6-14%)
-Endometrial Cancer (3%)
-Breast Cancer (12-25%)

Screening recommendations:
- Annual Colonoscopy age 25-30
- Upper endoscopy every 3m-2y age 30

Colectomy indications as per FAP except differ for prophylactic colectomy.

2015 ACG Guidelines
Therapeutic colectomy:
———————————-
Absolute indications:
- Documented colorectal cancer
- significant symptoms

Relative indications:
- multiple adenomas >6mm
- significant increase in adenoma numbers on serial exams
- adenoma with HGD
- inability to adequately survey colon due to multiple diminuitive polyps

Not generally recommended

How well did you know this?
1
Not at all
2
3
4
5
Perfectly
63
Q

How do you diagnose Serrated Polyposis syndrome?

A

Unlike other inherited CRC syndromes there is no clearly associated mutation to test for on genetic testing (RNF43 is closest association).

WHO 2019 criteria need either of:

  • ≥5 serrated polyps/lesions proximal to rectum with ≥5mm in size or 2 ≥10mm in size
    OR
  • 20+ serrated polyps/lesions in entire colon with ≥5 proximal to the rectum.

The polyp count is applied cumulatively and unfortunately it is not systematically applied and diagnosis is often missed especially due to different providers and may not have access to previous results.

How well did you know this?
1
Not at all
2
3
4
5
Perfectly
64
Q

What is the Hinchey classification?

A

Grading system for severity of diverticulitis to guide clinical management. Originally an intraoperative scoring system but modified to CT findings.

1 = pericolonic abscess <4cm treat w abx alone
2 = pelvic abscess or interloop abscess or abscess >4cm. Typically treat w drain percutaneous if amenable anatomically
3 = purrelent peritonitis and most likely need theatre
4 = faeculant peritonitis need theatre.

How well did you know this?
1
Not at all
2
3
4
5
Perfectly
65
Q

What constitutes complicated diverticulitis?

A
  • Perf, abscess, fistula, obstruction or stricture
  • In acute or chronic setting

Nb Phlegmon or extraluminal gas alone do not constitute complicated.

How well did you know this?
1
Not at all
2
3
4
5
Perfectly
66
Q

When is an elective segmental colectomy indicated for diverticular disease?

A
  • symptomatic affecting quality of life, multiple episodes per year, same or limited area of colon affected each time
  • stricture or partial obstruction

Generally however first episode is usually worse so if dealt with that ok may not need much intervention in future.

How well did you know this?
1
Not at all
2
3
4
5
Perfectly
67
Q

What is the recommended screening for those with serrated polyposis syndrome?

A
  • Colonoscopies every 1-3 years
  • Starting age not clearly defined.
  • No extracolonic manifestations to screen for.
How well did you know this?
1
Not at all
2
3
4
5
Perfectly
68
Q

This patient presents with dark red PR bleeding in your emergency department. Aside from the resuscitation and management of the bleeding. What do they likely have and what are the other manifestations/risks?

A

This is a classic hyperpigmented spots of the buccal mucosa and around the nose associated with Peutz-Jegher syndrome or a mutation in STK11/LKB1 genes. These spots are rarely present at birth but are pronounced by age 5 and can fade during puberty. They can also be around perianal skin and fingers.

These patients have high numbers of Hamartomatous polyps which are generally benign but can have a malignant transformation so at risk of colonic, stomach and small bowel cancers and they are also at risk of many other malignancies including breast, endometrial and cervical and benign ovarian tumours in women and sertoli cell testicular tumours in men. Also risk of pancreatic and lung ca.

Polyps arise in small bowel, colon, stomach and rectum in order of most frequent to less.

How well did you know this?
1
Not at all
2
3
4
5
Perfectly
69
Q

What do you recommend for screening for patients diagnosed with Peutz-Jegher syndrome?

A

Triple endoscopy (upper, lower and pill cam)
- start w baseline aged 8
- repeat every 1-3years.
- if no polyps found at index next repeat at 18yo and every 2-3y thereafter.

Breast MRI/USS/mamms starting age 30 w breast clinical exams from age 18

Pelvic exams
- testicular from age 10 +/- test USS
- pelvic exam and speculum age 18 annually

MRCP or EUS
- every 1-2y from age 30
Chest radiographs in smokers

How well did you know this?
1
Not at all
2
3
4
5
Perfectly
70
Q

During colonoscopy you remove 3 adenomatous polyps, 1 of them is 11mm the others are 3 and 4mm. What do you recommend for when they should have their next surveillance colonoscopy?

A

The adenoma >10mm means recommendation for next colonoscopy in 3 years according to the 2020 NZ MOH guidelines.

How well did you know this?
1
Not at all
2
3
4
5
Perfectly
71
Q

What are the NZ direct to colonoscopy referral criteria?

A

The NZ MOH referral criteria for direct access to outpatient colonoscopy are divided into 2 week category and 6 week category.

  • known or suspected CRC based on imaging or DRE to plan surgery or look for synchronous tumours.
  • IDA and PR bleeding unexplained by benign pathology.
  • > 50yo w altered bowel habit >6w + unexplained PR bleeding .

  • > 50yo w altered bowel habit >6w
  • 40-50yo w altered bowel habit >6w + unexplained PR bleeding.
  • > 50yo unexplained PR bleeding
  • Unexplained IDA
  • NZGuidelines Group (NZGG) Category 2 family hx + 1 or more of altered bowel habit >6w + unexplained PR bleeding and aged ≥40 years
  • NZGG Category 3 family hx + 1 or more of altered bowel habit >6w + unexplained PR bleeding and aged ≥25 years
  • imaging reveals >5mm polyp
  • suspected IBD

https://www.health.govt.nz/system/files/2019-02/referral-criteria-direct-access-outpatient-colonoscopy-computed-tomography-colonography-feb19-v2.pdf

How well did you know this?
1
Not at all
2
3
4
5
Perfectly
72
Q

What is the Dukes classification for colorectal cancer?

A

Applied to rectal and colon adenocarcinomas. No longer used as TNM staging favoured.

Stage A - CRC as far as muscularis propria, no nodal involvement

Stage B - CRC through wall as far as serosa, no nodal involvement.

Stage C - CRC involves nodes

Stage D - CRC has metastasised.

How well did you know this?
1
Not at all
2
3
4
5
Perfectly
73
Q

What are the Faecal occult blood tests available and their limitations.

A

There is the FIT test (faecal immunohistochemical test) assessing for presence of haemoglobin in the faeces. This is the one used by NZ bowel screening program and offered every 2 years to those aged 60-74.(or 50yo for Maori/Pacific Is in some areas of country).

There is the FOBT using Guaiac which reacts to peroxidase activity of haeme. many false negatives and false positives. Also will test postive if patient has eaten beef recently, vit C, iron supplementation.

There is a multi-targeted stool DNA test (Cologuard) which tests for molecular markers of CRC and is more sensitive but still low and optimal set of markers still to be developed.

How well did you know this?
1
Not at all
2
3
4
5
Perfectly
74
Q

What is CTC and what is its sensitivity?

A

CT colonography is a CT abdo/pelvis with bowel prep administered prior and then air insufflation and oral contrast given prior to scanning.

It is >90% sensitivity and specificity for polyps >10mm but drops significantly for polyps <5mm.

How well did you know this?
1
Not at all
2
3
4
5
Perfectly
75
Q

What is/are the serum tumour marker(s) used for colorectal cancer?

A

Carcinoembryonic antigen (CEA) is the most common.

Can also be expressed by pancreatic, stomach, breast, ovarian, lung, bladder and thyroid cancers

Can also be raised in many non-malignant states too including:
- smokers
- gastritis
- COPD
- fatty liver, cirrhosis or gallstones
- diverticulitis
- autoimmune conditions
- IBD

Used as marker of size or spread of known malignancy or highly suspected malignancy. Not used as a screening/diagnostic marker as not specific enough. Routinely performed once cancer diagnosed as pre-resection baseline then again 3months post resection and 3monthly thereafter for 5 years to assess for trends.

How well did you know this?
1
Not at all
2
3
4
5
Perfectly
76
Q

What are some good or poor prognostic features for colon cancer that you might see on the synoptic report?

A

Good prognosis features:
- no lymph node involvement
- low T stage
- no mets
- deficient MMR
- Microsatelite instability
- Lymphovascular penetration/reaction

Poor prognosis features
- lymph node involvement
- high T stage
- mets
- proficient MMR
- Microsatelite stable
- Lymphvascular invasion
- KRAS, NRAS or BRAF mutations

How well did you know this?
1
Not at all
2
3
4
5
Perfectly
77
Q

What is the TNM staging for colon cancer?

A

Stage 1-2 N0/M0
Stage 4 any T any N M1

How well did you know this?
1
Not at all
2
3
4
5
Perfectly
78
Q

You’ve excised an adenocarcinoma endoscopically. When would you say the excision is regarded complete vs need for segmental colectomy still required.

A

It is important to note that these decisions will be made in an MDM setting and taking into account patient wishes and comorbidities. However for an endoscopic excision to be regarded as complete (i.e. no lymphadenectomy will be performed) it has to be a low risk tumour taken in one piece (not piecemeal) with clear margins i.e. T1 and margins >2mm. It also has to be a well differentiated tumour with no lymphovascular invasion.

How well did you know this?
1
Not at all
2
3
4
5
Perfectly
79
Q

When is adjuvant chemotherapy recommended in colon cancer?

A

For stage 1 or 2:
- T4 disease
- Perforated disease
- Indeterminate or positive margins
- Poorly differentiated or undifferentiated
- Lymphvascular or perineural invasion
- Non-oncologic resection (<12nodes)

Stage 3 disease

Stage 4:
- Resectable or potentially resectable (i.e. isolated lung or liver) may consider neoadjuvant chemo
- isolated peritoneal mets (controversial)

in unresectable disease palliative chemo.

How well did you know this?
1
Not at all
2
3
4
5
Perfectly
80
Q

When would you consider neoadjuvant chemo in colon cancer?

A
  • T4b disease
  • Bulky lymph nodes
  • Potentially resectable tumour to downgrade to resectable
  • Mets
How well did you know this?
1
Not at all
2
3
4
5
Perfectly
81
Q

What is meant by R0, R1 and R2 resection?

A

R0 resect for cure.
R1 resect to microscopic residual disease
R2 resect to macroscopic residual disease

How well did you know this?
1
Not at all
2
3
4
5
Perfectly
82
Q

What are the typical chemotherapy agents for colorectal cancer?

A

FOLFOX (Folinic acid, 5 Flurouracil and Oxaloplatin)

CAPEOX (Capcitabine and Oxaloplatin)

FOLFIRI (Folinic acid, 5 Flurouracil, Irinotecan)

How well did you know this?
1
Not at all
2
3
4
5
Perfectly
83
Q

What is meant by the term ‘complete mesocolic excision’?

A

Resection of the cancer with clear margins (5cm) en bloc with its mesocolon with resection back to the primary vessel origin and including minimum nodal harvest of 12 lymph nodes.

How well did you know this?
1
Not at all
2
3
4
5
Perfectly
84
Q

In which appendiceal cancers would you proceed to a completion right hemicolectomy?

A
  • adenocarcinoma
  • mucinous if ruptured, incomplete margins or mod-poor differentiation
  • goblet cell
How well did you know this?
1
Not at all
2
3
4
5
Perfectly
85
Q

What is TME?

A

Total Mesorectal Excision is referring to quality sharp dissection (blunt increased local recurrence rate) of the mesorectum in totality without traversing in and out of the bloodless plane. Performed in a low anterior resection for rectal cancer.

86
Q

What curative resection options are available for caecal or ascending colon cancers?

A

Right hemicolectomy (lap or open)
- includes 7-10cm of TI
- Blood supply ligated ileocolic, right and middle colics.

87
Q

What curative resection options are available for hepatic flexure colon cancers?

A

Extended right hemicolectomy (lap or open)
- includes 7-10cm of TI
- includes proximal 1/2 of transverse colon
- Blood supply ligated ileocolic, right and middle colic

88
Q

What curative resection options are available for transverse colon cancers?

A

If very proximal transverse can treat with extended right.

Transverse colectomy (includes both hepatic and splenic flexures)
- middle colic ligated

If very distal transverse can treat with extended left.

89
Q

What curative resection options are available for descending colon cancers?

A

Left hemicolectomy
- ligated left branch of middle and left colics.

90
Q

What curative resection options are available for sigmoid colon cancers?

A

Sigmoid colectomy/ high anterior resection.
- Ligated portion of IMA

91
Q

What is a diverticulum?

A

Diverticula are saclike mucosal pouches that protrude from a tubular structure.

True diverticulum is projection of all layers of the GI tract wall. e.g. Zenkers, and Meckels

Pseudodiverticulum is projection of mucosa and submucosa through the muscular wall of the bowel e.g. colonic diverticulosis.

92
Q

What is the pathophysiology of colonic diverticular disease?

A

Colonic diverticulum are pseudodiverticula i.e. mucosa and submucosa sac like projections through the muscularis propria. They occur at sites of vasa recta arterial entrance into the muscularis propria and as such can be associated with diverticula bleeding.

Diverticular disease refers to a constellation of possible symptoms ranging from inflammation of the diverticulum, bleeding, interdiveritcula inflammation or SCAD (segmental colitis associated with diverticulosis) and abdominal pain.

The diverticula can occur anywhere along the colon (rectum spared as taenia colic splay here). Predominance of left sided/sigmoid diverticula in western population and right sided in Asian population likely pointing to different eitiology.

Risk factors include:
- ‘western diet’ i.e. low fibre, high fat and red meat.
- obesity
- sedentary lifestyle
- smoker

93
Q

What are some treatment options for a diverticular bleed?

A
  1. Watchful waiting - support them and if stable ~75% will settle spontaneously.
  2. Colonoscopy plus adjuncts adrenaline, clipping, diathermy
  3. CT-A localised the source and amenable and available in your institution then angioembolisation.
  4. Resection - if localised by CTA then can do segmental resection but if not then requires total colectomy.

Resection indicated if
- >72h of bleeding
- Transfusion dependent 4-6 u every 24h
- rebleeding on same admission

94
Q

Describe a way of classifying appendiceal neoplasms.

A

Mucinous and non-mucinois

Mucocoele
LAMNs
HAMNs
Mucinous adenocarcinomas

Epithelial tumours:
- Non-mucinous adenocarcinoma
- Goblet cell adenocarcicnoma

Neuroendocrine (NETs)

95
Q

What are mucinous appendiceal neoplasms?

A

Mucin filled luminal distension of the appendix accounting for ~1 in 200 appendicectomies.

Classified by Peritoneal Surface Oncology Group International (PSGI) into:

Simple mucocoele = degenerative changes in epithelium resulting in obstruction and dilatation of the appendix without dysplasia or hyperplasia. Produces mucin with no epithelial cells. Treatment - simple appendicectomy without rupture.

Mucinous neoplasms without infiltrative invasion:
- LAMN = low grade mucinous neoplasm
= basally oriented uniform nuclei with cytoplasm containing abundant mucin and little or no mitotic activity. Treatment - simple appendicectomy without rupture. If ruptured refer to peritonectomy centre as risk of pseudomyoxma peritonei (1/3-3/4 of patients).

  • HAMN = high grade mucinous neoplasm = cribriform growth, loss of polarity, enlarged nuclei, hyperchromatic nucleoli and increased mitotic rate. Treatment - simple appendicectomy without rupture. If ruptured refer to peritonectomy centre as risk of pseudomyoxma peritonei.
  • serrated lesions e.g. adenoma or sessile serated polyps can obstruct the appendix lumen causing mucocoele. Treatment = simple appendicectomy without rupture.

Mucinous neoplasms with infiltrative invasion:
Mucinous adenocarcinoma = neoplasm with >50% cross section comprised of extracellular mucin. Treatment - simple appendicectomy without rupture*. If ruptured refer to peritonectomy centre as risk of pseudomyoxma peritonei.
* no high grade evidence to suggest R hemi however most probably would as even with G1 well diff tumour still 5-6% risk of leaving positive lymph nodes behind if haven’t completed a R hemi and risks of major complications from R hemi in fit patient likely lower than this.

96
Q

How are LAMNs staged?

A

Despite not being a cancer still have a T staging system.

Tis if confined to appendiceal wall
T3 if through to subserosa
T4a if through to serosa

(no T1/2)

97
Q

How are HAMNs staged?

A

As per colorectal cancer T1-4.

98
Q

What are the histological features of serrated lesions?

A

Foci of proliferation in base of crypts and at irregular points along crypt lumen leading to asymmetric dilatation, horizontal growth of the crypts along the muscularis mucosa or exaggerated serrations that extend into the crypt accompanied by basilar dilatation.

99
Q

How are mucinous adenocarcinomas of the appendix classified?

A

TNM staging as well as

Differentiation
G1 Well
G2 Mod
G3 Poor

100
Q

What is the peritoneal carcinoma index?

A

A scoring system for peritoneal disease each of 9 regions of the abdominal wall and 4 segments of small bowel are each graded 0-3. A total score is calculated and combined with tumour characteristics, patients factors etc the peritonectomy centre will advise regarding if will benefit from CRS +/- HIPEC. As a general surgeon > take loads of photos.

101
Q

What surveillance do you recommend for mucinous neoplasms of the appendix post resection without rupture?

A

Simple Mucocoele - none

LAMN - CT CAP w po/IV contrast 6 monthyl for 2 years.

HAMN - CT CAP w po/IV contrast 4-6 monthly for 2 years then yearly.
+ tumour markers CEA, CA19-9, Ca125.

Serrated lesions - as per NZ colonoscopy guidelines

Adenocarcinomas - as per colorectal ca guidelines.

ALL need colonoscopy if not recent as up to 40% chance of syncrhonous lesion be it adenoma, tubulovillous adenoma, cancer etc.

102
Q

For non-mucinous adenocarcinoma and goblet cell carcinoma do you recommend appendicetomy or right hemicolectomy?

A

For T1s non-mucinous adenocarcinoma with high risk features or above then in appropriate patient should be getting right hemicolectomy For goblet cell carcinoma a right hemicolectomy.

103
Q

How are NETs of appendix classified?

A

By differentiation:

Well differentiated
G1 low grade
G2 mod grade
G3 high grade

Poorly differentiated = Neuroendocrine carcinomas (NECs)
small cell
large cell

Mixed neuroendocrine and non-neuroendocrine neoplasms (MiNENs)

104
Q

What is carcinoid syndrome?

A

https://dermnetnz.org/topics/carcinoid-syndrome

105
Q

How should NETs of appendix be worked up and treated?

A

If preoperative assessment shows <1cm proceed to appendicectomy

If >2cm proceed to R hemi colectomy but also CT CAP, baseline urinary 5 HIAA and Chromogranin A (CGA) (while off losec) +/- Dotatate CT.

All get colonoscopy.

Post op if
well differentiated/low risk features:
appendicectomy and no f/up

poorly differentiated or high grade or high risk features:
R hemicolectomy
3-6m CGAs/5-HIAA/CT CAP Dot

If metastatic:
Long standing somatostatin analogues or
mTOR inhibitors

106
Q

What is a Dotatate scan?

A

A PET CT (positron emission tomography) with gallilium 68 labelled somatostatin docks to somatostatin receptors on NETs. ~70-90% of NETs overexpress somatostatin receptors so hence this scan highlights where NETs and their mets are. In doing so can prognosticate how well a patient may respond to systemic somatostatin targeted chemotherapies as well.

107
Q

What are some possible causes of lower GI bleed?

A

Small Intestine
- Angiodysplasia
- Dieulafoy Lesion
- Meckel’s Diverticulum
- Intussusception
- Inflammatory Bowel Disease
- Radiation Enteritis
- Mesenteric Ischemia
- Aortoenteric Fistula
- Malignancy
- Recent Surgery

Large Intestine (85% of LGIB - Originate in the Colon)
-Angiodysplasia – Most -Common Cause of Obscure GI Bleeding
- Dieulafoy Lesion
- Stercoral Ulcer
- Diverticulosis – Most
Common Cause of GI Bleed - Inflammatory Bowel Disease
- Radiation Enteritis
- Infectious Colitis
- Ischemic Colitis
- Malignancy
- Recent Colonoscopy or Biopsy
- Recent Surgery

Rectum/Anus
- Angiodysplasia
- Dieulafoy Lesion
- Inflammatory Bowel
- Disease
- Radiation Proctitis
- Hemorrhoids
- Anal Fissure
- Anorectal Varices
- Local Trauma
- Malignancy
- Recent Surgery

108
Q

What is this?

A

Angiodysplasia of colon

Abnormal ectatic, dilated and thin walled blood vessels.
Most common cause of obscure GI bleeding.
Most commonly found in right colon.
Causes venous bleeding.
Can be associated with aortic stenosis, ESRD,Von willebrands.

Can treat endoscopically w haemostatic measures e.g. clips or coagulation, if refractory then resection or angiography. Can also consider octreotide and thalidomide…

109
Q

What is a Dieulafoys lesion?

A

Submucosal tortuous arteriole in GI tract. Can spontaneously rupture and bleed.

Most common site stomach followed by duodenum.

110
Q

What diagnostic criteria do you use for chronic constipation?

A

Rome IV.

≥ 2 Of:
- Straining During ≥ 25% of Bowel Movements
- Bristol Stool Scale #1-2 ≥ 25% of Bowel Movements
- Sensation of Incomplete Evacuation ≥ 25% of Bowel Movements
- Sensation of Anorectal Obstruction ≥ 25% of Bowel Movements
- Manual Maneuvers to Facilitate ≥ 25% of Bowel Movements
- < Three Bowel Movements Per Week
- Loose Stools Rarely Present without Laxative Use
- Insufficient Criteria for Irritable Bowel Syndrome (IBS)

111
Q

Define enteric fistula.

A

An abnormal connection between the bowel and another epithelialised viscus or skin.

112
Q

What are some risk factors for formation of an enteric fistula?

A

Trauma
Crohns
Faeculant peritonitis
Emergency procedure
Infection
Smoking
Malnutrition

113
Q

What are some factors that impede closure of an enteric fistula?

A

FRIENDS

Foreign body
Radiation expsoure
Inflammation
Epithelialisation
Neoplasia
Distal Obstruction
Steroids

114
Q

What are the Fischer stages of managment of enterocutaneous fistulas?

A
  1. Recognition and stabilisation (24h)
  2. Recognition and drainage of any abscess (24-48h)
  3. Nutritional assessment and supplementation 24-48h)
  4. Spontaneous closure <60days
  5. Operation
  6. Post operative care
  7. Maintain nutrition and start rehabilitation.
115
Q

What are the management principles of enterocutaenous fistulae?

A

SNAP

  • Skin protection and management of sepsis
  • Nutritional supplementation
  • Anatomy defined
  • Plan procedure
116
Q

Describe the intraoperative and endoscopic findings and macroscopic and microscopic findings associated with Crohns disease

A

Crohns is an inflammatory bowel disease that can affect anywhere along GIT.

Intraoperative:
- Fat creeping
- Bowel oedema
- Obstruction
- Fistulas
- Strictures

Endoscopic:
- Fissures and deep ulcers create Cobblestone appearance
- Serpinginous ulcers

Macroscopic:
- Transmural disease
- Skip lesions (discontinuous)

Microscopic:
- Non caseating granulomas

117
Q

What are the extraintestinal manifestations of Crohns?

A
  • Arthritis typically large
    joints e.g. SIJ
  • Skin - erythema nodosum and pyoderma gangrenosum
  • Eyes - uveitis, iritis, episcleritis
  • HPB - gallstone disease due to reduced resorption of bile salts in TI, and PSC although more typical of UC.
  • Renal stones (Ca Oxalate stones due to stetorrhoea > increased luminal binding of calcium > increased oxalate absorption in colon.
  • Haematological: Pernicious anaemia due to reduced B12 absorption in TI. Increased VTE risk.
118
Q

Describe the intraoperative and endoscopic findings and macroscopic and microscopic findings associated with Ulcerative colitis.

A

Ulcerative colitis is an autoimmune inflammatory bowel disease involving the mucosa and submucosa only.

Intraoperative:
- little to see as not transmural.
- in toxic megacolon would see large, distended colon +/- perforation

Endoscopic:
- Diffuse continuous ulceration and inflammation from rectum and spreading proximally to potentially involve the entire colon with potential for changes in terminal ileum as well due to backwash ileitis.
- Pseudopolyps.

Macroscopic:
- mucosal oedema
- mucosal ulceration circumferential
- pseudopolyps

Microscopic:
- Non granulomatous inflammation of mucosa and submucosa only
- Crypt abscess’
- Goblet cell hypoplasia.
- Polymorphonuclear cells in lamina propria

119
Q

What are the extraintestinal manifestations in common between Crohns and UC?

A

Both have the following:
Musculoskeletal: Arthritis usually affecting SIJ
Skin: Erythema nodosum and pyoderma gangrenosum
Eyes: Anterior uveitis, Episcleritis or Iritis
HPB: Both at risk of PSC though UC much more so.
Haematological: VTE risk

120
Q

Compare and contrast Crohns’ with UC.

A
121
Q

What is the Montreal classification used for Crohns?

A

Montreal score looks at age, location and behaviour or phenotype of the disease.

122
Q

How do you manage Crohns?

A

Management of acute flares to induce remission:
- 1st line corticosteroid
- admit to gastroenterology
- fluid resuscitation
- proactive VTE prophylaxis
- low res diet
- avoid antidiarrhoeals as can precipitate toxic megacolon.
> immunosuppressive agentts e.g. mesalazine or azathioprine or biologics such as TNFa inhibitors inflixumab

Management to maintain remission:
- azathioprine
- smoking cessation
- low res diet and nutritional support
- IBD MDT
- colonoscopic surveilance from 10y post diagnosis to assess for CRC.

Management of extraintestional manifestations and psychological aspects.

123
Q

What is the Truelove Witt classification system for Ulcerative Colitis?

A

Combines clinical values including HR, # of BMs/24, bloody stools and fever and laboratory values including Hb <105 and ESR >30 to assess the severity of the Ulcerative Colitis during an acute flare.

Easy to use, lab values easy to obtain except CRP in may institutions is preferred to ESR.

124
Q

What is the Mayo DAI scoring system for Ulcerative Colitis?

A

Used to assess the disease activity index or severity of the Ulcerative Colitis over time and to assess treatment response. It is one of the most common scoring systems used in UC trials.

Combines clinical assessment (stool freq, blood in stool, and physicians assessment*) with endoscopic findings (mild-severe) each scores 0-3.

*Physicians assessment encompasses general patient wellbeing, abdominal pain, functional status and extraintestinal manifestations. This of course has an element of subjectivity.

Also note for stool frequency and rectal bleeding, the patient acts as their own point of reference (i.e. number of stools above normal) rather than the absolute number.

125
Q

What is the Montreal score for UC?

A

A scoring system evaluating the extent (E) and severity (S). The Severity is based off Truelove Witt.

126
Q

What is the UCEIS score for UC?

A

Endoscopic evaluation of severity. Looks at 3 factors: vascular pattern (0-2), bleeding (0-3), ulceration (0-3).

Interpretation
0-1 =Remission
2-4 = Mild
5-6 = Moderate
7-8 = Severe

127
Q

What is the disease activity index used for Crohn’s?

A

Combines clinical values including weight loss, abdominal pain, abdo mass, # of soft stools, # of bloody stools and extraintestinal manifestations (above is essentailly the Harvery-Bradshaw index) with use of antidiarrhoeal medications and lab values Hct.

CDAI <150 is a marker of remission of Crohn’s disease.
CDAI >450 is a marker of severe Crohn’s disease.

128
Q

Which extraintestinal manifestations improve for UC patients post proctocolectomy?

A

Manifestations that Improve:
- Erythema Nodosum
- Arthritis
- Eye Problems

Manifestations that Improve for Some (50%):
- Pyoderma Gangrenosum

Manifestations that Do Not Improve:
- Primary Sclerosing Cholangitis
- Ankylosing Spondylitis

129
Q

Describe the dynamics of defecation.

A
  1. Colon peristalsis and mass muscle movements expel faeces from colon into rectum
  2. Rectal distension triggers stretch receptors
  3. Activates defecation reflex which forces faeces towards anus.
  4. Inhibitory myenteric plexus signals relax the internal anal sphincter.
  5. External anal sphincter under voluntary control relaxes to release faeces.

https://www.ncbi.nlm.nih.gov/books/NBK539732/

130
Q

What is a rectocoele and what are the risk factors?

A

Weakened rectovaginal septum to herniation of rectal wall thoruhg posterior vaginal wall.

They are graded as small <2.5cm, mod 2-4cm, and large >4cm.

Risk factors is vaginal childbirth particularly if traumatic or prolonged and increased age.

131
Q

What is the pathogenesis of diverticulosis?

A

Not well understood.

However several factors at play include colonic wall structure, motility, dietary fibre, genetics, vit D levels, obesity and physical activity have been thought to influence its development.

One theory for low fibre diet fosters longer transit times and requires higher colonic pressures to move material through however more controversial. However does appear that a low fibre diet increases chance of complications of diverticular disease.

132
Q

Describe the pathogenesis of colorectal cancer.

A

Colorectal cancer develops through a multistep process involving the accumulation of genetic and epigenetic changes. Key mutations occur in oncogenes like KRAS and tumour suppressor genes like APC and TP53, leading to uncontrolled proliferation, evasion of apoptosis, induction of angiogenesis and loss of cell cohesion can lead to progression from benign polyps to malignant tumours, and potential metastasis.

Environmental factors, such as diet and inflammation, can also contribute to its pathogenesis likely through epigenetic influences.

133
Q

Spot diagnosis and pathogenesis

A

This is showing pseudomembranous colitis.

It is most commonly driven by clostridium difficile (a gram positive bacillus, spore forming anaerobe) infection following antibiotics and imbalance in gut microflora.

Clostridium difficile produces Exotoxins A and B. Leads to damage of mucosa, drives inflammation cascade with oedema and extensive neutrophil infiltration in lamina propria. Pseudomembranes are comprised of bacteria, fibrin, mucous and neutrophils from intercrypt erosions.

Can lead to SIRS and sepsis.

134
Q

How would you work up and manage this patient?

A

This is pseudomembranous colitis most commonly caused by C.diff.

Work up and management with simultaneous assessment and resuscitation using CRISP algorithm.

Investigations include:
Lab: FBC, U&E, Cr, Lactate and albumin
Stool spec: M/C/S, c.diff toxin
Imaging: AXR, CT Abdo/pelvis
Endoscopy: May have diagnosis on flexi sig but 1/3 cases limited to right colon so may need colonoscopy.

Management:
- Correct fluid and electrolyte imbalances
- Contact isolation
- Cease offending antibiotics
- Oral metronidazole 400mg TDS for 14 days
- monitor for complications
- avoid antiperistaltic agents
- consult ID to exclude resistant strains
- cholestyramine mya be useful in refractory cases to bind toxin.
Surgery reserved for:
- toxic megacolon
- fulminant disease (severe SIRS/sepsis)
- colonic perforation
- failure of medical therapy in 48-72h w continued sign of toxicity.
- once decision made to operate it must be performed in expeditious and aggressive manner with total colectomy with preservation of rectum and end ileostomy even if segmental colitis. *NB exterior may appear normal at time of surgery as is a mucosa disease.

If no improvement 2nd line is vancomycin oral +/- rectal if ileus or toxic megacolon.

  • for refractory or recurrent disease consider faecal microbiome transplant in consultation with ID.
135
Q

What is the hypervirulent strain of C.Diff?

A

NAP1/BI/027

136
Q

What are the antibiotics most commonly associated with causing C.diff?

A

Clindamycin
Fluroquinolones e.g. Ciprofloxacin
Cephalosporins

137
Q

What are some risk factors for C.diff development?

A
  • Antibiotics (clinda, cipro, ceph)
  • age >65
  • prolonged hospital stay +/- ICU stay
  • severe comorbid illness
  • F > M
  • gastric acid suppression
  • enteral feeding
  • GI surgery
  • IBD
  • chemotherapy
  • obesity
138
Q

Define toxic megacolon.

A

Segmental or pancolonic dilatation >/= 6cm (or 12cm in caecum) in the presence of acute colitis and systemic toxicity.

139
Q

Describe the pathogenesis of toxic megacolon.

A

Toxic megacolon involves severe colonic inflammation usually triggered by ulcerative colitis or infection e.g. c,diff and subsequent dilatation of the colon proximal to the inflammed segment mediated through damaged smooth muscle release of nitrous oxide leading to colonic wall atony and dysmotility. Systemic SIRS/sepsis also occurs largely due to translocation of bacteria and resultant bacteraemia.

140
Q

How is toxic megacolon diagnosed?

A

Jalan criteria:
- radiographic evidence of colonic dilatation (>6cm or caecum >12cm)
+ 3 of:
- fever >38C
- HR >120bpm
- Neutrophil leukocytosis >10.5
- Anaemia
+ >/= 1 of:
- dehydration
- altered sensorium
- electrolyte disturbances
- hypotension

141
Q

What is the management of toxic megacolon?

A

50% will succeed with medical therapy, 50% will need surgery.

Medical management:
- Gut rest (NBM, NGT, IVF)
- steroids if due to IBD, inflixumab if steroid resistant
- broadspectrum abx

Surgical management indicated if:
- perforation
- peritonitis
- abdominal compartment syndrome
- uncontrolled haemorrhage
- severe SIRS/sepsis
- ongoing severe or worsening disease despite medical therapy after 48-72h

Operation would be subtotal colectomy and end ileostomy formation.

142
Q

What are differentials for colitis?

A

Vascular - Ischaemic acute vs chronic
Infectious - bacterial, viral, parasitic
Autoimmune - IBD UC, crohns, indeterminate, behcets disease
Iatrogenic - Diversion
Drug related e.g. chemotherapy neutropenic enterocolitis, NSAID induced

Other
Amyloidosis
Collagenous colitis

143
Q

What are some common pathogens in infectious enteritis and colitis? How are they treated?

A

Gram negative rods:
- Campylobacter - cipro or erythro
- E.Coli esp O157:H7, shiga-like toxin. can also cause haemolytic uraemic syndrome, resolve on own, avoid abx can make it worse
- Salmonella - none usually, if thyphi then 3rd gen ceph
- Shigella - as for salmonella
- Yersinia - 3rd gen ceph

Gram positive rod:
- Actinomyces israelli
- C.Diff

Mycobacterium tuberculosis

Viral:
- CMV most common in AIDS/immunosuppressed. (on edocscopic biospy would see large atypical cells with intranuclear inclusion bodies)
- Norovirus
- Rotavirus
- Adenovirus

Parasitic:
- Entomoeba histolytica (see liver abscess’ too. been to Mexico?)
- Giardia

144
Q

What is the pathogenesis of ischaemic colitis?

A

Can be acute or chronic and non occlusive/ low flow states or occlusive eitiology.

Non-occlusive states:
- sepsis
- haemorrhagic shock
- hypotension
- MI
- haemodialysis
- cardiopulmonary bypass
- medications e.g. opioids

vs occlusive:
thrombi
emboli

most common sites are watershed areas i.e. splenic flexure and upper rectum.

145
Q

How is ischaemic colitis diagnosed?

A

CT may be suggestive but not definitive. Arterial phase may see occlusive event e.g.thrombus or embolus.

Colonoscopy is gold standard as long as no indication for straight to surgery.
- no bowel prep
- advance until distal extent of disease, do not traverse.
- minimise insufflation

146
Q

What are indications for operative management of ischaemic colitis?

A
  • gangrenous bowel
  • perforation
  • massive haemorrhage
  • peritonitis
  • unstable
  • sepsis
147
Q

What is diversion colitis and what is the pathogenesis of it?

A

Inflammation of a defunctionalised- bypassed segment of colon after surgical diversion.

Due to short chain fatty acid e.g. butyrate deficiency (primary colonocytes’ nutrient)

It is a diagnosis of exclusion

148
Q

What are the medical management options for Ulcerative Colitis?

A

Induce remission:
- Steroids
- Immune modulators e.g. azathioprine, mercaptopurine, methotrexate
- Biologics e.g. TNFa inhibitors e.g. inflixumab and adalimumab (Humira)
- Infection control: cipro/metronidazole

Maintain remission:
- 5-ASA (aminosalicylic acid) oral and or rectal

148
Q

How is diversion colitis managed?

A

By reanastomosis where/when possible

If not possible Short chain fatty acid enemas or steroids or aminosalicylic acids (5-ASA)

149
Q
A
150
Q

What is the risk of developing CRC with UC?

A

Ulcerative colitis patients have a six times greater risk of developing colorectal cancer than those of average risk. But, that being said, only about 5% of people with severe ulcerative colitis will end up developing this type of cancer.

151
Q

What endoscopic techniques or strategies should be considered when assessing ulcerative colitis patients at colonoscopy?

A

Chromendoscopy - dye spray or Virtual chromendoscopy.

High resolution colonoscopy

Targeted biopsies and non targeted biopsies every 10cm from 4 spaced areas in standard white light colonoscope (but only targeted necessary in skilled chromendoscopy) of areas previously affected by colitis or dysplasia.

Surveillance should begin 8-10 years after onset of symptoms or immediately upon diagnosis of PSC. They should be repeated every 1-5 years depending on personalised risk recalculated at each colonoscopy. Post total proctocolectomy the pouch still requires surveillance endoscopy.

152
Q

What is the modified Paris classification for endoscopic lesion appearance?

A

Divides lesions into polypoid and non-polypoid and invisible.

Polypoid includes peduculated and sessile

Non-polypoid includes flat-elevated, flat and flat-depressed.

Invisible is dysplastic lesion that is picked up on random biopsy from an area of normal appearing. mucosa.

153
Q

What are indications for total colectomy +/- procto and IPAA in UC?

A

Emergent :
- toxic megacolon
- perforation
- massive GI bleed
- refractory fulminant colitis.

Elective:
- medical intractability
- obstruction
- malignancy
- A finding of unresectable visible dysplasia or of invisible multifocal or high-grade dysplasia
- failure to thrive in children

154
Q

What is the pathogenesis of appendicitis?

A

Primarily caused by luminal obstruction and stasis e.g. appendicolith, impacted stool, polyp, appendiceal tumour or caecal tumour.

> appendiceal wall oedema and mucus secretion
increased luminal and intramural pressures
small vessel thrombosis and lymphatic stasis
impaired blood flow and venous congestion and endothelial cell damage, ischaemia and necrosis
bacterial infection and translocation
SIRS and sepsis

155
Q

What are the possible positions of the appendix relative to the caecum and ileum?

A

Retrocaecal (32-62%)
Paracolic/ precaecal
Post ileal
Pre ileal
Pelvic
Subcaecal

156
Q

Describe why appendicitis classically presents with migratory pain.

A

Initially starts with luminal obstruction > distension and this distension causes pain signal due to stretch receptors in appendiceal wall and given appendix is part of mid gut embryologically it is felt by visceral nerves T8-T10 and is poorly localisable and felt centrally. As the appendicitis progresses and the inflammation and infection become transmural and begin to irritate the overlying parietal peritoneum the somatic nerves are now signalling pain and these are far more localised to Mc Burneys point.

157
Q

What features on USS would be suggestive of appendicitis?

A

Appendiceal diameter >6mm
Wall distension >3mm
Appendix not compresible
Appendicolith
Periappendiceal fat stranding and fluid

158
Q

What diagnostic scoring system do you use for appendicitis?

A

Alvarado (MANTRELS)
Migratory to RLQ (1)
Anorexia (1)
Nausea and vomiting (1)
Tenderness in RLQ (2)
Rebound tenderness (1)
Elevated temperature (1)
Leukocytosis WCC>10 (2)
Shift of neutrophils (1)

Low Scores (0-3): Evaluate Other Etiologies (No CT Indicated)
Intermediate Scores (4-6): CT Scan
High Scores (7-10): Surgical Consultation

Low Scores Are Better to “Rule-Out” Appendicitis than High Scores Are to “Rule-In”
Score < 3-4 to “Rule-Out” Appendicitis Has 96% Sensitivity
Score > 6-7 to “Rule-In” Appendicitis Has 58-88% Sensitivity

159
Q

In what patients would you chose to do an interval appendicectomy and at what timeframe?

A

Rationale for interval appendicectomy is due to:
- recurrence rate up to ~40%
- risk of malignancy in the specimen of an interval appendicectoym for previously perforated appendicitis ~11%
- if interval appx forgone then CT and colonoscopy for those >40yo.

Disease factors:
Complicated appendicitis with phelgmon/abscess may treat with antibiotics and interval appendicectomy at 6-8 weeks unless patient unstable/ refractory to abx then may need to operate acutely. May need perc drain for abscess >3-4cm.

Patient factors:
Pregnancy
Recent medical event making anaesthesia too risky e.g. MI.
concerns on imaging for more than just an appendicitis e.g. ?underlying malignancy may delay, to allow further workup and colonoscopy etc first.

160
Q

Describe how you would approach each of these scenarios. You are performing a diagnostic lap for suspected appendicitis and find:
1. normal appearing appendix, no other pathology grossly visible
2. messenteric adenitis
3. fat creeping along terminal ileum
4. friable appendiceal base
5. ovarian cyst >5cm
6. ectopic pregnancy
7. ovarian torsion

A
  1. Depends on how patient was counselled prior regarding this possibility. My current practice is to recommend in this instance that the appendix is removed to prevent future diagnostic laps for possible recurrent symptoms.
  2. My current practice is to leave appendix alone. Have explanation for symptoms.
  3. Leave appendix alone. Take lots of photos. Consult Gastroenterology post op.
  4. Staple across limited caecectomy being mindful to protect ileocaecal valve.
  5. O&G consult and possible cystectomy
  6. O&G consult and salpingectomy
  7. Detort the ovary and O&G consult. Generally rarely need oophorectomy even if ducky and black looking. Only if falls apart or associated malignancy do need to do oopherectomy.
161
Q

Discuss pros and cons of antibiotics treatment alone for uncomplicated appendicitis.

A

Advantages:
- Most respond clinically
- faster return to work
- no increased perforation rate
- ~90% are able to avoid surgery at initial admission

Disadvatages:
- high recurrence rate
- 30% require appendicectomy within 90days, 40% within 1st year, 50% within first 5 years.
- 28x higher risk of complications (largely those with appendicolith) study by CODA Collab in 2020 therefore contraindicated in those with faecolith.

https://pubmed.ncbi.nlm.nih.gov/33017106/

162
Q

Discuss appendicitis in pregnancy.

A
  • pregnant women less likely than non-pregnant women to develop appendicitis
  • in those with appendicitis they’re more likely to perforate than non-pregnant women ?diagnostic delay or hesitation to operate
  • most common non-obstetric indication for surgery during pregnancy.

First trimester:
- most common cause of acute abdominal pain
- risk of miscarriage

Second trimester:
- Most frequent trimester
- Safest time to operate

Third trimester:
- Lowest overall rate of appendicitis
- Most likely to perforate

In uncomplicated appendicitis:
- fetal loss ~2%
- preterm labour ~6%

In complicated appendicitis:
- fetal loss ~8%
- preterm labour ~11%

Only 50-60% Have a Classical Clinical Presentation
May Instead Present with Heartburn, Flatulence, or Diarrhoea
Location of Pain:
Most Commonly at McBurney’s Point Regardless of Trimester.
May Migrate to the Right Mid-Upper Quadrant in the Third Trimester Due to a Gravid Uterus.
Leukocytosis May Be Normal in Pregnancy.

Treatment:
Uncomplicated Appendicitis: - Appendectomy
Optimal Approach Remains Inconclusive
- Laparoscopic Associated with Lower Overall Complications and Shorter Length of Stay
- Laparoscopic Associated with Higher Rate of Fetal Loss (Odds Ratio 1.82)
- Similar Rates of Preterm Labour

Open Incision: Generally Recommended to Make at McBurney’s Point or the Site of Maximal Pain (Possibly More Cephalad)

Complicated Appendicitis:
Phlegmon/Abscess: Debated
Poor Evidence to Guide Decision Making
Consider Antibiotics with Percutaneous Drainage and Interval Appendectomy (Similar to Nonpregnant Patients) vs Immediate Appendectomy
Free Perforation: Open Appendectomy

Laparoscopic Trocar Placement: First Trimester
Normal Placement
Second Trimester
Access: Hasson (May Consider Veress at Subxiphoid or Left Costal Margin)
10 mm Port: Supraumbilical
5 mm Ports (x2): LLQ & RLQ
Third Trimester
Access: Hasson (Consider Veress Subxiphoid or Left Costal Margin)
10 mm Port: Supraumbilical
5 mm Ports (x2): Two Along Right Side
RLQ, Right Mid-Abdomen, RUQ or Subxiphoid
Also Consider Positioning in Full or Partial Left Lateral Decubitus During the Second/Third Trimesters

163
Q

What is a treatment ladder for pseudoobstruction?

A

Pseudoobstruction is colonic dilatation without mechanical obstruction.

<10-12cm caecum then bowel rest
- NBM
- NGT
- serial AXR and exam
- cease opiates and consider naloxone

> 10-12cm caecum or above fails after 48-72h
- neostigmine
(needs HDU bed as can cause bradycardia)

> if neostigmine CI or fails then endoscopic decompression +/- placement of flatus tube.

> if above doesn’t work caecostomy tube.

Surgery indications are for ischaemia or perforation.

164
Q

What are the risks of colonoscopy?

A

Major bleeding 8 in 10,000
Perforation 4 in 10,000
Missed pathology
Sedation associated
Prep associated - dehydration, light headedness, vomiting

165
Q

When would you consider a colonic stent?

A

Temporising measure in obstructing or very near obstructing colon cancer as a bridge to elective surgery in a centre well versed at performing colonic stents.

This allows for preoperative optimisation, bowel preparation and reduce need for emergency surgery which is known to be higher risk for many complications.

Demonstrated to be useful for some left sided cancers but jury out for right sided cancers still. Also the optimal timing post stent placement is still debated though most recommend within 2 weeks.

166
Q

What are some complications of colonic stents?

A

Stent migration
Reobstruction
Perforation (nb converts to T4 disease)

167
Q

What is short course vs long course radiotherapy in rectal cancer?

A

Short course is 25Gy given over 1 week and is just radiotherapy followed by surgery in 1 week.

Long course is 50Gy given over 5 weeks with radiosensitising chemotherapy usually oral capcitabine. Followed by surgery 8-12 weeks.

No difference between long and short in local recurrence rate, disease free survival, overall survival or APR rate however is higher rate of downstaging and higher rate of complete pathological response in long course but also higher rate of toxicity

In choosing which approach consider patient factors include regional location ability to access radiotherapy and disease factors.

168
Q

Who benefits from neoadjuvant radiation in rectal cancer?

A

DisTANCE
Distance - low/close to anus
T >3
Anal sphincter involvement
Nodal disease
CRM threatened or involved (w/n 1mm)
Extramural vascular invasion

169
Q

What is TNT?

A

Total Neoadjuvant Treatment is combination of radiotherapy and chemotherapy regime all given upfront i.e. the traditional neoadjuvant regime + the traditional adjuvant regime but all given upfront.

Can be given as induction chemotherapy where chemo is first or consolidation chemo given after the chemorads or rads treatment.

e.g.
Long course > FOLFOX or FOLFIRINOX etc > Surgery
Short course > FOLFOX > Surgery
FOLFOX > Long course > Surgery
FOLFOX > Short course > Surgery

170
Q

What is optimal timing for resection post neoadjuvant treatment for rectal cancer?

A

Still debated. Priniciples are the longer you wait the better the local response to the radiation however after 12 weeks it makes surgery more challenging and also longer you wait the longer it has to metastasise.

Generally accepted to wait between 8-12 weeks ideally for long course or TNT treatment but for short course alone then 1 week.

171
Q

What are some classification systems used at time of endoscopy for polyps?

A

NICE
Paris
Kudo pit pattern

https://www.binasss.sa.cr/feb24/12.pdf

172
Q

What are the possible complications in radiation therapy in rectal cancer?

A
173
Q

What is low anterior resection syndrome?

A
174
Q

Define radiation proctitis

A

Acute: direct radiation injury to the rectal mucosa which results in oedema, ulceration and shedding of mucosa.

Chronic: Injury to small blood vessels, progressive ischaemic changes, atrophy, telegectasia, strictures and fistulae.

175
Q

How do you treat radiation proctitis?

A

Medical
- hydration
- dietary changes
- antidiarrhoeals
- analgesia
- antispasmodics
- oral/rectal steroids or formalin enemas or butyrate enemas.

Endoscopic
- APC

Surgical

176
Q

What are the benefits of Trans rectal ultrasound (TRUS) vs MRI for rectal cancer?

A

TRUS is superior in assessing early T stage (T1/2) cancers.
Excellent delineation between a tumour, the mucosa and the muscularis propria and the tumour extension beyound the muscuaris propria.

MRI is superior to TRUS for assessment of CRM, gives larger field of view for more detail about more proximal tumours. Tends to be less operator dependent and technique dependent. Allows assessment of obstructing cancers which TRUS does not. Gives more info regarding iliac nodes.

177
Q

Classify colonic polyps

A

Neoplastic vs non neoplastic and mucosal vs submucosal.

Neoplastic mucosal polyps:
- benign adenomatous (tubular, vilous, tubulovilous)
- serrated
- adenoma with intramucosal invasive carcinoma (in situ)
- adenoma with invasive carcinoma

Non-neoplastic mucosal polyps:
- hamartomas
- Peutz Jegher polyp
- metaplastic/hyperplastic polyps
- inflammatory polyps

Neoplastic submucosal polyps:
- lipomatous
- leiomyomatous
- lymphomatous
- carcinoid
- GIST

Non-neoplastic submucosal
- lymphoid aggregates

178
Q

How can you classify adenomatous polyps?

A

Morphologically:
Pedunculated vs sessile

Histologically:
Tubular - dysplastic tubules in >80% of lesion

Villous - dysplastic villous fronds in >80% of lesion (NB the finger like vili are actually elongated crypts)

Tubulovillous - >20% tubular, <80% villous

179
Q

What is the significance of tubular, villous or tubulovillous classification for adenomatous polyps?

A

Risk of cancer is greatest in villous subtype. For polyps <10mm risk of malignancy with tubular is 1%, but 10% for villous. When polyps are larger the risk of malignancy is high irrespective of histological type.

180
Q

What is the cause of adenomatous polyps?

A

Multifactorial in aeitiology inclduing genetic predisposition (e.g. germ line APC mutation, mismatch repair gene mutations), environemental factors (e.g. dietary, smoking,) and increases with age.

181
Q

What is the relationship between polyps and CRC development?

A

Stepwise accumulative damage to the DNA results in loss of cell cohesion, loss of tumour suppression ability, increased angiogenesis, increased mitotic rate.

Seen as progressive proliferation then dysplasia then adenocarcinoma.

e.g. APC gene tumour suppressor gene (TSG) mutation is an early event, followed by k-ras oncogene mutation, then DCC (TSG) mutation followed by p53 (TSG)

182
Q

How does a mutation in APC gene result in increased risk of CRC?

A

APC (adenomatous polyposis coli) gene located on chromosome 5 encodes a cytoplastic protein responsible for beta-catenin regulation. In its mutated form it leads to unregulated concentration of beta-catenin which in turn upregulates cell cycling leading to unchecked proliferation. Using the 2 hit hypothesis this is the first ‘hit’ and subsequent hits to tumour suppressor genes or oncogenes lead to accumulative step wise change from normal epithelium to dysplasia to adenocarcinoma as the cells acquire an ability to evade apoptosis, increase in numbers, promote angiogenesis and loose cell to cell cohesion.

Although a germline mutation in APC (resulting in FAP) is only responsible fro 1% of CRC, 80% of sporadic CRCs also harbour truncation mutations in APC gene.

183
Q

What are the macroscopic and microscopic features of carcinoid tumours?

A

Macroscopic:
- nodular growth in the mucosa and submucosa
- bright yellow appearance once fixed in formalin
- often a mesenteric desmoplastic reaction

Microscopic:
- solid masses of monotonous-appearing cells with small round nuclei
- typically have moderate amount of finely granular cytoplasm and fine nucleoli.
- derived from enterochromaffin-like cells of the neuroendocrine system of the GI tract so stain for chromogranin A or synaptophysin.

184
Q

In an acute LBO, what factors are going to influence what management decision you make? And what are the general options available?

A

Patient factors
- are they going to heal a join e.g. vasculopath, smoker, morbidly obese, frail, malnourished

Disease factors
- synchronous tumours, underlying eitiology, location and extent of tumour/obstruction

Surgeon factors
- capability, time of day

Institution factors
- equipment available, expertise available e.g. stents

Intraoperative factors
- caecum quality
- mets
- tumour invasion e.g. into adjacent organs etc
- perforation

Options available:
- stenting
- diversion e.g. loop colostomy, loop ileostomy
- resection +/- anastomosis +/- covering ileostomy
- bypass
- palliation

185
Q

What percentage of FAP patients have a family history of same?

A

75%

so 25% are the first patient in their family tree with it.

186
Q

What is Spigelmans scoring system?

A

Score for duodenal polyps in FAP. Looks at 4 factors and determines frequency of duodenal endoscopy.

  • size
  • number of polyps
  • histology
  • dysplasia
187
Q

What are the familial CRC syndromes? How could you classify them?

A
188
Q

What are the subtypes of FAP?

A
189
Q

How is FAP diagnosed?

A

Suspected for:
- >10 adenomas cumulative <30yo
- >20 adenomas cumulative over lifetime
- Family history with AD inheritance pattern
- colorectal adenomas with pattern of extracolonic manifestations attributable to FAP.

Confirm with germline mutation in APC

190
Q

What are the extracolonic manifestations of FAP?

A

Causes of mortality in FAP are CRC, duodenal/periampullary adenocarcinomas, desmoids.

191
Q

What would you recommend for surveillance colonoscopy for different polyp scenarios. List some.

A

file:///Users/bex/Downloads/Update-on-polyp-surveillance-guidelines-2020-NBSP-edits-2024.pdf

192
Q

What are the surgical options for someone with an acutely obstructing distal colon cancer?

A
193
Q

What is the ‘watch and wait’ referring to and who is it used for?

A

It is an emerging therapeutic option for select individuals with rectal cancer. Essentially in those who have undergone neoadjuvant chemoradiation treatment an intensive highly protocolised follow up program involving MRI pelvis, flexi sigi and DREs is followed assessing for those patients with complete clinical response to attempt organ preservation.

Some studies suggest up to 50% of patients obtain organ preservation with this.

Needs to be performed in a centre familiar and comfortable with this and with experienced colorectal surgeons and good follow up protocols.

Clinical features of cCR
- normal DRE
- flat white scar
- telengectasia
- (negative biopsies not mandatory)

MRI features of cCR
- no residual tumour
- residual fibrosis
- no nodal disease

194
Q

What are the types of caecal volvulus?

A

Axial
Loop
Caecal bascule

195
Q

In the pathogenesis of colorectal cancer what are the 3 described pathways?

A

Chromosomal instability pathway (~85% of CRC cases)

Chromosomal instability pathway (CIN) , CpG island methylator phenotype pathway (CIMP) and the pure Microsatellite instability (MSI) pathway from germline mutations in MMR genes (HNPCC).

  • APC > KRAS >DCC>tp53
  • *Remember ‘A Krazy Disease Process’
  • Classical adenoma > carcinoma sequence
  • Only a very small minority of CRCs characterised by CIN possess a full complement of these molecular abnormalities. It is possible that several of these clonal events can be bypassed by other genetic or epigenetic aberrations in order to deliver the necessary biological consequences.
  • APC = TSG, mutation = FAP
  • KRAS = oncogene
  • DCC/SMAD4 ch18. - Juvenille Polyposis syndrome
  • tP53 - TSG, Li Fraumeni
    -originate from adenomatous polyps
  • microsatellites are nucleotide repeat sequences scattered throughout the genome that don’t encode genes.
  • MSI refers to a discrepancy (and thus instability)
  • Mismatch repair (MMR) dysfunction results in MSI in
    the number of these repeats.
  • mutations in MLH-1, MSH-2, MSH-6 and PMS-2 have all been implicated in HNPCC
  • originate from adenomatous polyps
  • Whilst HNPCC causes the pure form of MSI, the majority of MSI-H CRCs occur sporadically in the context of DNA methylation of the MLH1 promoter and the consequent transcriptional silencing of MLH1 expression. Such cancers exhibit both CIMP and MSI, and are considered here as part of the CIMP pathway
  • second most common pathway
  • epigenetic silencing of tumour suppressor genes (through methylation of promoter region of genes)
  • there is not yet one universally endorsed panel of CIMP markers or even a gold-standard technique for characterising methylation for the diagnosis of CIMP
  • strong association with BRAF, KRAS and MLH-1 methylation.
  • characterised by a well defined cluster of clinicopathological features, including proximal location and a gender and age bias for the development of CIMP in older women.
  • sessile serrated adenomas are the chief pathological precursor
  • Better prognosis by stage
    but often present later stage often missed at colonoscopy
  • don’t respond as well to 5FU chemo
196
Q

What are the histological subtypes of colorectal cancer?

A

Listed in most common to rarer.

  • Adenocarcinoma
  • Mucinous adenocarcinoma (>50% tumour volume = extracellular mucin)
  • Signet ring (>50% cells are signets, poor prognosis)
  • Medullary (sheets of epitheliod neoplastic cells, pushing border, good prognosis).
197
Q

What surveillance do you recommend for those with a strong family history of colorectal cancer but no known germline mutation?

A

Based on NZ MOH July 2023 guidelines:

Category 1 =
NBSP

Category 2 =
- colo every. 5 years from age 50 OR
- 10y before earliest age of family members age at diagnosis of CRC.
(whichever comes first)
- Then provided high quality colo they go to NBSP from age 60 or based on previous polyps guidelines.

Category 3 =
- Refer to NZ Familial GI cancer service
- follow the plan from NZFGCS

198
Q

What are the family history categories?

A

Category 1 =
- 1 1st degree relative diagnosed with CRC
>/= 55 yo
- Termed individuals with slightly above average risk.

Category 2 =
- 1 1st -degree relative diagnosed with CRC < 55 yo OR
- 2 1st -degree relatives on the same side of the family diagnosed with CRC
at any age (without any of the potential high-risk features in Category 3).
- Termed individuals with moderately increased risk.

Category 3 =
- Termed individuals with potentially high risk
- one or more of the following:
- FHx of CRC syndromes
- 1 1st -degree relative + 2 or more 1st or 2nd degree relatives with CRC all on same side of family at any age.
- 2 1st-degree relatives, or 1 1st-degree relative + 1 or more 2nd -degree relatives, all on the same side of the family with a diagnosis of CRC, and 1
such relative:
* was diagnosed with CRC <55yo OR
* developed 2 or more bowel cancers, OR
* developed an extracolonic tumour suggestive of Lynch syndrome (ie, endometrial,
ovarian, stomach, small bowel, renal pelvis, pancreas or brain)
- a first-degree relative with CRC diagnosed under the age of 50, where colorectal
tumour immunohistochemistry has revealed loss of protein expression for one of the
mismatch repair genes (ie, MLH1, MSH2, MSH6 or PMS2) and further testing (BRAF
or methylation) raises the possibility of Lynch syndrome
- at least 1 1st- or 2nd-degree family member diagnosed with CRC in
association with multiple bowel polyps (10 or more adenomas at one time or 5 or more
advanced adenomas at one time, or 20 cumulative adenomas, 10 cumulative
adenomas if the patient is aged 30 years or younger) or a polyposis syndrome
* a 1st-degree relative with multiple colonic polyps (10 or more adenomas at one time
or 5 or more advanced adenomas at one time, or 20 cumulative adenomas, or 10
cumulative adenomas if the relative is aged 30 years or younger).

199
Q

When do you bowel prep for elective bowel resections?

A

CARD NEEDS UPDATING

The MOBILE2 Randomized Clinical Trial published in JAMA March 2024 reduced overall postoperative complications as well as rates of SSIs and anastomotic dehiscences in patients undergoing elective rectal resection compared with MBP plus placebo
https://jamanetwork.com/journals/jamasurgery/fullarticle/2816726

200
Q

In brief discuss the role of blood transfusion in patients with colorectal cancer.

A

Patients with colorectal cancer often have accompanying anaemia or perioperative bleeding.

Allogeneic transfusion may become necessary in these cases.

Some studies have found that perioperative transfusion could suppress the immune function and increase the recurrence and metastasis but others have not.

If the anaemia is not lifethreatening and there is time to wait for the effect, often an iron transfusion is preferred.

https://wjso.biomedcentral.com/articles/10.1186/s12957-018-1551-y#:~:text=Patients%20with%20colorectal%20cancer%20often,not%20%5B3%2C%204%5D.

201
Q

Define TME.

A

It is the standard surgical technique of sharp dissection in the embryological mesocolic plane to resect the rectal cancer en bloc with its mesorectal lymphovascular package to reduce local recurrence rates.

202
Q

Where do you want to ligate the IMA for a rectal cancer resection?

A

Ideally 1cm distal to its origin off the aorta. This allows for a small cuff and control of bleeding but gives maximal lymphovascular harvest and allows for greatest mobility of the remaining colon for anastomosis.

203
Q

What are accepted margins for rectal cancer?

A

5cm proximal and 2cm distal and 1mm CRM.

If below the distal mesorectal margin a 1cm margin is often accepted but less than this the patient needs an APR.

204
Q

When do you harvest lateral pelvic lymph nodes for rectal cancer?

A

Some Japanese groups recommend lateral wall lymph node dissection in T3/4 rectal cancers

In NZ in practice may perform targets lymph node dissection of pathological appearing nodes on imaging pre-operatively but not routinely performed.

205
Q

When might you consider an APR for rectal cancer?

A

Disease factors:
- rectal cancer is too low to obtain a 1cm + distal margin
- invading sphincter complex
- recurrent disease

Patient/functional factors:
- poor sphincter function preoperatively e.g. prev birth trauma
- patient preference

206
Q

What is the role for a covering ileostomy?

A

Reduces sequelae of anastomotic leak doesn’t reduce the risk of the leak.

Think how likely is patient to leak and how well would they tolerate a leak. Considered in patients with:

Disease factors:
- low join <6cm from anal verge

Operative factors:
- emergency operation
- significant blood loss
- instability
- anastomosis under tension
- positive leak test

Patient factors:
- preop radiation
- PVD, liver disease, renal, immunosuppressed.
- malnutrition

207
Q

When do you reverse a covering ileostomy?

A

It depends on disease and patient factors.

Disease
- what other adjuvant treatment is planned
- what is their prognosis

Patient
- have they recovered from their index surgery
- are they well nourished
- how are they coping with the ileostomy outputs, are they high, are there complications of the ileostomy
- is there an anastomotic leak on imaging

Early = 8-12 days
Late = 2months

208
Q

Tell me about role for colonic pouches.

A

Better functional outcomes for first 12 months, but don’t continue after 12months and given increases length and complexity of surgery and length of the bowel required to be brought down and can therefore increase tension on the anastomosis, colonic pouches are rarely performed.

209
Q

What is the arc of Riolan?

A

The meandering messentery artery is an anastomotic artery between SMA and IMA.

210
Q

What is Griffith point?

A

The point of collateralisation of SMA and IMA. It is the watershed area between the left colic and marginal artery of drummond occuring in region of splenic flexure around 2/3rd along transverse colon.

211
Q

What is Sudek point?

A

The point of collateralisation between the IMA and internal iliac artery. I.e. the watershed area at rectosigmoid junction. The last sigmoidal branch of the IMA usually forms and anastomosis with a branch of the superior rectal artery.